Programs & Examples On #Digg

0

try/catch blocks with async/await

Alternative Similar To Error Handling In Golang

Because async/await uses promises under the hood, you can write a little utility function like this:

export function catchEm(promise) {
  return promise.then(data => [null, data])
    .catch(err => [err]);
}

Then import it whenever you need to catch some errors, and wrap your async function which returns a promise with it.

import catchEm from 'utility';

async performAsyncWork() {
  const [err, data] = await catchEm(asyncFunction(arg1, arg2));
  if (err) {
    // handle errors
  } else {
    // use data
  }
}

Error: Module not specified (IntelliJ IDEA)

For IntelliJ IDEA 2019.3.4 (Ultimate Edition), the following worked for me:

  1. Find the Environment variables for your project.
  2. Specify, from the dropdowns, the values of "Use class path of module:" and "JRE" as in the attached screenshot.

enter image description here

configuring project ':app' failed to find Build Tools revision

In my case issue was with wrong path to NDK.

If your project requires it please check it in menu File -> Project Structure -> SDK Location:

enter image description here

How does one set up the Visual Studio Code compiler/debugger to GCC?

There is a much easier way to compile and run C code using GCC, no configuration needed:

  1. Install the Code Runner Extension
  2. Open your C code file in Text Editor, then use shortcut Ctrl+Alt+N, or press F1 and then select/type Run Code, or right click the Text Editor and then click Run Code in context menu, the code will be compiled and run, and the output will be shown in the Output Window.

Moreover you could update the config in settings.json using different C compilers as you want, the default config for C is as below:

"code-runner.executorMap": {
    "c": "gcc $fullFileName && ./a.out"
}

How to fix symbol lookup error: undefined symbol errors in a cluster environment

yum update

helped me out. After I had

wget: symbol lookup error: wget: undefined symbol: psl_latest

OperationalError, no such column. Django

I faced this problem and this is how I solved it.

1) Delete all the migration records from your app's migration directory. These are files named 0001_,0002_,0003_ etc. Be careful as to not delete the _init__.py file.

2) Delete the db.sqlite3 file. It will be regenerated later.

Now, run the following commands:

python manage.py makemigrations appname
python manage.py migrate

Be sure to write the name of your app after makemigrations. You might have to create a superuser to access your database again. Do so by the following

python manage.py createsuperuser

Swift Beta performance: sorting arrays

Swift 4.1 introduces new -Osize optimization mode.

In Swift 4.1 the compiler now supports a new optimization mode which enables dedicated optimizations to reduce code size.

The Swift compiler comes with powerful optimizations. When compiling with -O the compiler tries to transform the code so that it executes with maximum performance. However, this improvement in runtime performance can sometimes come with a tradeoff of increased code size. With the new -Osize optimization mode the user has the choice to compile for minimal code size rather than for maximum speed.

To enable the size optimization mode on the command line, use -Osize instead of -O.

Further reading : https://swift.org/blog/osize/

http://localhost:8080/ Access Error: 404 -- Not Found Cannot locate document: /

I think I figured out the questions after reading the log. Thanks to Will's reminder, I checked the log and found out the some program else is listening to that port. Before I can start to figure out which program, my computer was restarted and localhost:8080 works and showing tomcat page. Whooh

how to set mongod --dbpath

mongod  --port portnumber --dbpath /path_to_your_folder

By default portnumber is 27017 and path is /var/lib/mongodb

You can set your own port number and path where you want to keep all your database.

Apache: "AuthType not set!" 500 Error

Remove the line that says

Require all granted

it's only needed on Apache >=2.4

Using the RUN instruction in a Dockerfile with 'source' does not work

I had the same problem and in order to execute pip install inside virtualenv I had to use this command:

RUN pip install virtualenv virtualenvwrapper
RUN mkdir -p /opt/virtualenvs
ENV WORKON_HOME /opt/virtualenvs
RUN /bin/bash -c "source /usr/local/bin/virtualenvwrapper.sh \
    && mkvirtualenv myapp \
    && workon myapp \
    && pip install -r /mycode/myapp/requirements.txt"

I hope it helps.

Decoding and verifying JWT token using System.IdentityModel.Tokens.Jwt

Within the package there is a class called JwtSecurityTokenHandler which derives from System.IdentityModel.Tokens.SecurityTokenHandler. In WIF this is the core class for deserialising and serialising security tokens.

The class has a ReadToken(String) method that will take your base64 encoded JWT string and returns a SecurityToken which represents the JWT.

The SecurityTokenHandler also has a ValidateToken(SecurityToken) method which takes your SecurityToken and creates a ReadOnlyCollection<ClaimsIdentity>. Usually for JWT, this will contain a single ClaimsIdentity object that has a set of claims representing the properties of the original JWT.

JwtSecurityTokenHandler defines some additional overloads for ValidateToken, in particular, it has a ClaimsPrincipal ValidateToken(JwtSecurityToken, TokenValidationParameters) overload. The TokenValidationParameters argument allows you to specify the token signing certificate (as a list of X509SecurityTokens). It also has an overload that takes the JWT as a string rather than a SecurityToken.

The code to do this is rather complicated, but can be found in the Global.asax.cx code (TokenValidationHandler class) in the developer sample called "ADAL - Native App to REST service - Authentication with ACS via Browser Dialog", located at

http://code.msdn.microsoft.com/AAL-Native-App-to-REST-de57f2cc

Alternatively, the JwtSecurityToken class has additional methods that are not on the base SecurityToken class, such as a Claims property that gets the contained claims without going via the ClaimsIdentity collection. It also has a Payload property that returns a JwtPayload object that lets you get at the raw JSON of the token. It depends on your scenario which approach it most appropriate.

The general (i.e. non JWT specific) documentation for the SecurityTokenHandler class is at

http://msdn.microsoft.com/en-us/library/system.identitymodel.tokens.securitytokenhandler.aspx

Depending on your application, you can configure the JWT handler into the WIF pipeline exactly like any other handler.

There are 3 samples of it in use in different types of application at

http://code.msdn.microsoft.com/site/search?f%5B0%5D.Type=SearchText&f%5B0%5D.Value=aal&f%5B1%5D.Type=User&f%5B1%5D.Value=Azure%20AD%20Developer%20Experience%20Team&f%5B1%5D.Text=Azure%20AD%20Developer%20Experience%20Team

Probably, one will suite your needs or at least be adaptable to them.

npm install errors with Error: ENOENT, chmod

Be careful with invalid values for keys "directories" and "files" in package.json

If you start with a new application, and you want to start completely blank, you have to either start in a complete empty folder or have a valid package.json file in it.

If you do not want to create a package.json file first, just type: npm i some_package

Package with name "some_package" should be installed correctly in a new sub folder "node_modules".

If you create a package.json file first, type: npm init Keep all the defaults (by just clicking ENTER), you should end up with a valid file.

It should look like this:

{
  "name": "yourfoldername",
  "version": "1.0.0",
  "description": "",
  "main": "index.js",
  "scripts": {
    "test": "echo \"Error: no test specified\" && exit 1"
  },
  "author": "",
  "license": "ISC"
}

Note that the following keys are missing: "directories", "repository" and "files". It looks like if you use incorrect values for "directories" and/or "files", you are not able to install the package. Leaving these keys out, solved the issue for me.

Also note key "main". This one is present, but it does contain an invalid value. No file "index.js" exists (yet). You can safely remove it.

Now type: npm i some_package and package with name "some_package" should be installed correctly in a new sub folder "node_modules".

Excel VBA Code: Compile Error in x64 Version ('PtrSafe' attribute required)

I think all you need to do for your function is just add PtrSafe: i.e. the first line of your first function should look like this:

Private Declare PtrSafe Function swe_azalt Lib "swedll32.dll" ......

Why does the preflight OPTIONS request of an authenticated CORS request work in Chrome but not Firefox?

This is an old post but maybe this could help people to complete the CORS problem. To complete the basic authorization problem you should avoid authorization for OPTIONS requests in your server. This is an Apache configuration example. Just add something like this in your VirtualHost or Location.

<LimitExcept OPTIONS>
    AuthType Basic
    AuthName <AUTH_NAME>
    Require valid-user
    AuthUserFile <FILE_PATH>
</LimitExcept>

CKEditor automatically strips classes from div

Another option if using drupal is simply to add the css style that you want to use. that way it does not strip out the style or class name.

so in my case under the css tab in drupal 7 simply add something like

facebook=span.icon-facebook2

also check that font-styles button is enabled

How to solve privileges issues when restore PostgreSQL Database

To solve the issue you must assign the proper ownership permissions. Try the below which should resolve all permission related issues for specific users but as stated in the comments this should not be used in production:

root@server:/var/log/postgresql# sudo -u postgres psql
psql (8.4.4)
Type "help" for help.

postgres=# \du
               List of roles
    Role name    | Attributes  | Member of
-----------------+-------------+-----------
 <user-name>    | Superuser   | {}
                 : Create DB
 postgres       | Superuser   | {}
                 : Create role
                 : Create DB

postgres=# alter role <user-name> superuser;
ALTER ROLE
postgres=#

So connect to the database under a Superuser account sudo -u postgres psql and execute a ALTER ROLE <user-name> Superuser; statement.

Keep in mind this is not the best solution on multi-site hosting server so take a look at assigning individual roles instead: https://www.postgresql.org/docs/current/static/sql-set-role.html and https://www.postgresql.org/docs/current/static/sql-alterrole.html.

creating array without declaring the size - java

As others have said, use ArrayList. Here's how:

public class t
{
 private List<Integer> x = new ArrayList<Integer>();

 public void add(int num)
 {
   this.x.add(num);
 }
}

As you can see, your add method just calls the ArrayList's add method. This is only useful if your variable is private (which it is).

how to get yesterday's date in C#

Something like this should work

var yesterday = DateTime.Now.Date.AddDays(-1);

DateTime.Now gives you the current date and time.

If your looking to remove the the time element then adding .Date constrains it to the date only ie time is 00:00:00.

Finally .AddDays(-1) removes 1 day to give you yesterday.

Force DOM redraw/refresh on Chrome/Mac

An approach that worked for me on IE (I couldn't use the display technique because there was an input that must not loose focus)

It works if you have 0 margin (changing the padding works as well)

if(div.style.marginLeft == '0px'){
    div.style.marginLeft = '';
    div.style.marginRight = '0px';
} else {
    div.style.marginLeft = '0px';
    div.style.marginRight = '';
}

Using number as "index" (JSON)

JSON is "JavaScript Object Notation". JavaScript specifies its keys must be strings or symbols.

The following quotation from MDN Docs uses the terms "key/property" to refer to what I more often hear termed as "key/value".

https://developer.mozilla.org/en-US/docs/Web/JavaScript/Data_structures#Objects

In JavaScript, objects can be seen as a collection of properties. With the object literal syntax, a limited set of properties are initialized; then properties can be added and removed. Property values can be values of any type, including other objects, which enables building complex data structures. Properties are identified using key values. A key value is either a String or a Symbol value.

How to start working with GTest and CMake

You can get the best of both worlds. It is possible to use ExternalProject to download the gtest source and then use add_subdirectory() to add it to your build. This has the following advantages:

  • gtest is built as part of your main build, so it uses the same compiler flags, etc. and hence avoids problems like the ones described in the question.
  • There's no need to add the gtest sources to your own source tree.

Used in the normal way, ExternalProject won't do the download and unpacking at configure time (i.e. when CMake is run), but you can get it to do so with just a little bit of work. I've written a blog post on how to do this which also includes a generalised implementation which works for any external project which uses CMake as its build system, not just gtest. You can find them here:

Update: This approach is now also part of the googletest documentation.

How to format a URL to get a file from Amazon S3?

Its actually formulated more like:

https://<bucket-name>.s3.amazonaws.com/<key>

See here

Android and Facebook share intent

Here is something I did which open Facebook App with Link

shareIntent = new Intent(Intent.ACTION_SEND);
shareIntent.setComponent(new ComponentName("com.facebook.katana",
                    "com.facebook.katana.activity.composer.ImplicitShareIntentHandler"));

shareIntent.setType("text/plain");
shareIntent.putExtra(Intent.EXTRA_TEXT,  videoUrl);

Oracle - How to create a materialized view with FAST REFRESH and JOINS

The key checks for FAST REFRESH includes the following:

1) An Oracle materialized view log must be present for each base table.
2) The RowIDs of all the base tables must appear in the SELECT list of the MVIEW query definition.
3) If there are outer joins, unique constraints must be placed on the join columns of the inner table.

No 3 is easy to miss and worth highlighting here

How does python numpy.where() work?

How do they achieve internally that you are able to pass something like x > 5 into a method?

The short answer is that they don't.

Any sort of logical operation on a numpy array returns a boolean array. (i.e. __gt__, __lt__, etc all return boolean arrays where the given condition is true).

E.g.

x = np.arange(9).reshape(3,3)
print x > 5

yields:

array([[False, False, False],
       [False, False, False],
       [ True,  True,  True]], dtype=bool)

This is the same reason why something like if x > 5: raises a ValueError if x is a numpy array. It's an array of True/False values, not a single value.

Furthermore, numpy arrays can be indexed by boolean arrays. E.g. x[x>5] yields [6 7 8], in this case.

Honestly, it's fairly rare that you actually need numpy.where but it just returns the indicies where a boolean array is True. Usually you can do what you need with simple boolean indexing.

REST API - why use PUT DELETE POST GET?

This is a security and maintainability question.

safe methods

Whenever possible, you should use 'safe' (unidirectional) methods such as GET and HEAD in order to limit potential vulnerability.

idempotent methods

Whenever possible, you should use 'idempotent' methods such as GET, HEAD, PUT and DELETE, which can't have side effects and are therefore less error prone/easier to control.

Source

How to obtain the absolute path of a file via Shell (BASH/ZSH/SH)?

Here's a zsh-only function that I like for its compactness. It uses the ‘A’ expansion modifier — see zshexpn(1).

realpath() { for f in "$@"; do echo ${f}(:A); done }

Logging framework incompatibility

SLF4J 1.5.11 and 1.6.0 versions are not compatible (see compatibility report) because the argument list of org.slf4j.spi.LocationAwareLogger.log method has been changed (added Object[] p5):

SLF4J 1.5.11:

LocationAwareLogger.log ( org.slf4j.Marker p1, String p2, int p3,
                          String p4, Throwable p5 )

SLF4J 1.6.0:

LocationAwareLogger.log ( org.slf4j.Marker p1, String p2, int p3,
                          String p4, Object[] p5, Throwable p6 )

See compatibility reports for other SLF4J versions on this page.

You can generate such reports by the japi-compliance-checker tool.

enter image description here

Set Page Title using PHP

I know this is an old post but having read this I think this solution is much simpler (though technically it solves the problem with Javascript not PHP).

    <html>
    <head>
    <title>Ultan.me - Unset</title>
    <script type="text/javascript">
        function setTitle( text ) {
            document.title = text;
        }
    </script>
    <!-- other head info -->
    </head>

    <?php 
    // Make the call to the DB to get the title text. See OP post for example
    $title_text = "Ultan.me - DB Title";

    // Use body onload to set the title of the page
    print "<body onload=\"setTitle( '$title_text' )\"   >";

    // Rest of your code here
    print "<p>Either use php to print stuff</p>";
    ?>
    <p>or just drop in and out of php</p>
    <?php

    // close the html page
    print "</body></html>";
    ?>

PHP function to generate v4 UUID

A slight variation on Jack's answer to add support for PHP < 7:

// Get an RFC-4122 compliant globaly unique identifier
function get_guid() {
    $data = PHP_MAJOR_VERSION < 7 ? openssl_random_pseudo_bytes(16) : random_bytes(16);
    $data[6] = chr(ord($data[6]) & 0x0f | 0x40);    // Set version to 0100
    $data[8] = chr(ord($data[8]) & 0x3f | 0x80);    // Set bits 6-7 to 10
    return vsprintf('%s%s-%s-%s-%s-%s%s%s', str_split(bin2hex($data), 4));
}

Super-simple example of C# observer/observable with delegates

Something like this:

// interface implementation publisher
public delegate void eiSubjectEventHandler(eiSubject subject);

public interface eiSubject
{
    event eiSubjectEventHandler OnUpdate;

    void GenereteEventUpdate();

}

// class implementation publisher
class ecSubject : eiSubject
{
    private event eiSubjectEventHandler _OnUpdate = null;
    public event eiSubjectEventHandler OnUpdate
    {
        add
        {
            lock (this)
            {
                _OnUpdate -= value;
                _OnUpdate += value;
            }
        }
        remove { lock (this) { _OnUpdate -= value; } }
    }

    public void GenereteEventUpdate()
    {
        eiSubjectEventHandler handler = _OnUpdate;

        if (handler != null)
        {
            handler(this);
        }
    }

}

// interface implementation subscriber
public interface eiObserver
{
    void DoOnUpdate(eiSubject subject);

}

// class implementation subscriber
class ecObserver : eiObserver
{
    public virtual void DoOnUpdate(eiSubject subject)
    {
    }
}

. observer pattern C# with event . link to the repository

Is there a pretty print for PHP?

I use this function for debugging:

function pre($pre=true) {
    if($pre) {
        echo "<pre>";
    }
    foreach(func_get_args as $arg) {
        print_r($arg);
        # or, if it pleases you more:
        # var_dump(arg); 
    }
    if($pre) {
        echo "<pre>";
    }

}

This way you don't have to

pre($arrayOne);
pre($arrayTwo);

But in one go:

pre($arrayOne, $arrayTwo); 

Or as many arguments you give it.

Where is the default log location for SharePoint/MOSS?

SharePoint uses a lot of different logging mechanisms. Most importantly you can configure the location of the logs through Central Admin. To give you an understanding of the logs involved, here is a quote from http://raiumair.wordpress.com/2007/06/19/quick-a-to-z-of-sharepoint-logs/

All file based logs can be read by text editors and can be parsed by using popular log parsing tools (Log Parser 2.2 from Microsoft or Funnel Web). It will also be a good idea to read the IIS Logs which are generally saved at (System Drive):\WINDOWS\system32\LogFiles

a) Diagnostics Logs

· Event Throttling Logs – These end up going to the Windows Event Log and can be viewed in the Event Viewer. They show Errors and Warnings.

· Trace Logs – These show detailed line by line tracing infomration emitted during a web request or service execution. They end up being stored at a known location on the front-end server. Default Location: (System Drive):\Program Files\Common Files\Microsoft Shared\Web Server Extensions\12\LOGS\

b) Audit Logs - They end up in the associated Content Database tables and can be viewed at Site Collection Level as well as Site Level using the web browser. WSS 3.0 and MOSS 2007 use different pages to show Audit Log Reports.

c) Usage Logs – They get stored locally on the front-end servers and get processed both locally and at farm level via SSP (this is based on the setup as I understand the results from the local processing are merged by SSP) and can be viewed at both the Site Level and Site Collection Level. Default Location: (System Drive):\Program Files\Common Files\Microsoft Shared\Web Server Extensions\12\Logs

d) Search\Query Logs – These are saved in the associated SSP database but can be viewed at SSP level via the Web Browser and in MOSS at Site Collection Level by going to the settings page.

e) Information Management Logs – Stored in the associated Content Database and can be can be viewed at the Site Collection Level.

f) Content and Structure Logs – This option is only available after one enables the publication feature. This store is saved in the Content Database associated with the Site Collection and can be viewed at Site Collection level by going to the settings page.

os.walk without digging into directories below

This is how I solved it

if recursive:
    items = os.walk(target_directory)
else:
    items = [next(os.walk(target_directory))]

...

git-upload-pack: command not found, when cloning remote Git repo

Matt's solution didn't work for me on OS X, but Paul's did.

The short version from Paul's link is:

Created /usr/local/bin/ssh_session with the following text:

#!/bin/bash
export SSH_SESSION=1
if [ -z "$SSH_ORIGINAL_COMMAND" ] ; then
    export SSH_LOGIN=1
    exec login -fp "$USER"
else
    export SSH_LOGIN=
    [ -r /etc/profile ] && source /etc/profile
    [ -r ~/.profile ] && source ~/.profile
    eval exec "$SSH_ORIGINAL_COMMAND"
fi

Execute:

chmod +x /usr/local/bin/ssh_session

Add the following to /etc/sshd_config:

ForceCommand /usr/local/bin/ssh_session

How can the error 'Client found response content type of 'text/html'.. be interpreted

I had got this error after changing the web service return type and SoapDocumentMethod.

Initially it was:

[WebMethod]
public int Foo()
{
    return 0;
}

I decided to make it fire and forget type like this:

[SoapDocumentMethod(OneWay = true)]
[WebMethod]
public void Foo()
{
    return;
}

In such cases, updating the web reference helped.

To update a web service reference:

  • Expand solution explorer
  • Locate Web References - this will be visible only if you have added a web service reference in your project
  • Right click and click update web reference

How do the likely/unlikely macros in the Linux kernel work and what is their benefit?

In many linux release, you can find complier.h in /usr/linux/ , you can include it for use simply. And another opinion, unlikely() is more useful rather than likely(), because

if ( likely( ... ) ) {
     doSomething();
}

it can be optimized as well in many compiler.

And by the way, if you want to observe the detail behavior of the code, you can do simply as follow:

gcc -c test.c objdump -d test.o > obj.s

Then, open obj.s, you can find the answer.

How to display activity indicator in middle of the iphone screen?

If you want to try to do it by using interface, you can view my video for this. This way, you no need to write any code to show Activity Indicator in the middle of the iPhone screen.

How to iterate object keys using *ngFor

You have to create custom pipe.

import { Injectable, Pipe } from '@angular/core';
@Pipe({
   name: 'keyobject'
})
@Injectable()
export class Keyobject {

transform(value, args:string[]):any {
    let keys = [];
    for (let key in value) {
        keys.push({key: key, value: value[key]});
    }
    return keys;
}}

And then use it in your *ngFor

*ngFor="let item of data | keyobject"

File size exceeds configured limit (2560000), code insight features not available

This is built from Álvaro González's answer and How to increase IDE memory limit in IntelliJ IDEA on Mac?

Go to Help > Edit Custom Properties

Add:

idea.max.intellisense.filesize=999999

Restart the IDE.

AngularJS : automatically detect change in model

In views with {{}} and/or ng-model, Angular is setting up $watch()es for you behind the scenes.

By default $watch compares by reference. If you set the third parameter to $watch to true, Angular will instead "shallow" watch the object for changes. For arrays this means comparing the array items, for object maps this means watching the properties. So this should do what you want:

$scope.$watch('myModel', function() { ... }, true);

Update: Angular v1.2 added a new method for this, `$watchCollection():

$scope.$watchCollection('myModel', function() { ... });

Note that the word "shallow" is used to describe the comparison rather than "deep" because references are not followed -- e.g., if the watched object contains a property value that is a reference to another object, that reference is not followed to compare the other object.

Maximum on http header values?

I also found that in some cases the reason for 502/400 in case of many headers could be because of a large number of headers without regard to size. from the docs

tune.http.maxhdr Sets the maximum number of headers in a request. When a request comes with a number of headers greater than this value (including the first line), it is rejected with a "400 Bad Request" status code. Similarly, too large responses are blocked with "502 Bad Gateway". The default value is 101, which is enough for all usages, considering that the widely deployed Apache server uses the same limit. It can be useful to push this limit further to temporarily allow a buggy application to work by the time it gets fixed. Keep in mind that each new header consumes 32bits of memory for each session, so don't push this limit too high.

https://cbonte.github.io/haproxy-dconv/configuration-1.5.html#3.2-tune.http.maxhdr

How can I loop through enum values for display in radio buttons?

Two options:

for (let item in MotifIntervention) {
    if (isNaN(Number(item))) {
        console.log(item);
    }
}

Or

Object.keys(MotifIntervention).filter(key => !isNaN(Number(MotifIntervention[key])));

(code in playground)


Edit

String enums look different than regular ones, for example:

enum MyEnum {
    A = "a",
    B = "b",
    C = "c"
}

Compiles into:

var MyEnum;
(function (MyEnum) {
    MyEnum["A"] = "a";
    MyEnum["B"] = "b";
    MyEnum["C"] = "c";
})(MyEnum || (MyEnum = {}));

Which just gives you this object:

{
    A: "a",
    B: "b",
    C: "c"
}

You can get all the keys (["A", "B", "C"]) like this:

Object.keys(MyEnum);

And the values (["a", "b", "c"]):

Object.keys(MyEnum).map(key => MyEnum[key])

Or using Object.values():

Object.values(MyEnum)

Is there a way to create key-value pairs in Bash script?

In bash, we use

declare -A name_of_dictonary_variable

so that Bash understands it is a dictionary.

For e.g. you want to create sounds dictionary then,

declare -A sounds

sounds[dog]="Bark"

sounds[wolf]="Howl"

where dog and wolf are "keys", and Bark and Howl are "values".

You can access all values using : echo ${sounds[@]} OR echo ${sounds[*]}

You can access all keys only using: echo ${!sounds[@]}

And if you want any value for a particular key, you can use:

${sounds[dog]}

this will give you value (Bark) for key (Dog).

Protractor : How to wait for page complete after click a button?

You don't need to wait. Protractor automatically waits for angular to be ready and then it executes the next step in the control flow.

How to Convert a Text File into a List in Python

Maybe:

crimefile = open(fileName, 'r')
yourResult = [line.split(',') for line in crimefile.readlines()]

SQL SERVER, SELECT statement with auto generate row id

Is this perhaps what you are looking for?

select NEWID() * from TABLE

Simplest way to detect a mobile device in PHP

There is no reliable way. You can perhaps look at the user-agent string, but this can be spoofed, or omitted. Alternatively, you could use a GeoIP service to lookup the client's IP address, but again, this can be easily circumvented.

blur() vs. onblur()

I guess it's just because the onblur event is called as a result of the input losing focus, there isn't a blur action associated with an input, like there is a click action associated with a button

How do you get the string length in a batch file?

@echo off & setlocal EnableDelayedExpansion
set Var=finding the length of strings
for /l %%A in (0,1,10000) do if not "%Var%"=="!Var:~0,%%A!" (set /a Length+=1) else (echo !Length! & pause & exit /b)

set the var to whatever you want to find the length of it or change it to set /p var= so that the user inputs it. Putting this here for future reference.

Conda command not found

MacOSX: cd /Users/USER_NAME/anaconda3/bin && ./activate

ActiveX component can't create object

The app is trying to create a COM Object and even if that COM DLL exists, it may depend on another DLL which isn't installed. You can use DependencyWalker to find out if this is the case.

Calling constructors in c++ without new

Quite simply, both lines create the object on the stack, rather than on the heap as 'new' does. The second line actually involves a second call to a copy constructor, so it should be avoided (it also needs to be corrected as indicated in the comments). You should use the stack for small objects as much as possible since it is faster, however if your objects are going to survive for longer than the stack frame, then it's clearly the wrong choice.

CSS: background-color only inside the margin

I needed something similar, and came up with using the :before (or :after) pseudoclasses:

#mydiv {
   background-color: #fbb;
   margin-top: 100px;
   position: relative;
}
#mydiv:before {
   content: "";
   background-color: #bfb;
   top: -100px;
   height: 100px;
   width: 100%;
   position: absolute;
}

JSFiddle

No Persistence provider for EntityManager named

Put the "hibernate-entitymanager.jar" in the classpath of application.
For newer versions, you should use "hibernate-core.jar" instead of the deprecated hibernate-entitymanager

If you are running through some IDE, like Eclipse: Project Properties -> Java Build Path -> Libraries.

Otherwise put it in the /lib of your application.

Insert variable values in the middle of a string

I would use a StringBuilder class for doing string manipulation as it will more efficient (being mutable)

string flights = "Flight A, B,C,D";
StringBuilder message = new StringBuilder();
message.Append("Hi We have these flights for you: ");
message.Append(flights);
message.Append(" . Which one do you want?");

How to read lines of a file in Ruby

Ruby does have a method for this:

File.readlines('foo').each do |line|

http://ruby-doc.org/core-1.9.3/IO.html#method-c-readlines

ORA-12514 TNS:listener does not currently know of service requested in connect descriptor

Starting the OracleServiceXXX from the services.msc worked for me in Windows.

How to redirect DNS to different ports

Possible solutions:

  1. Use nginx on the server as a proxy that will listen to port A and multiplex to port B or C.

  2. If you use AWS you can use the load balancer to redirect the request to specific port based on the host.

What is an Android PendingIntent?

Pending Intent is an intent who provides all permission to other application to do a particular works. When the main activity is destroyed, Android OS takes back the permission from it.

How to get the current user in ASP.NET MVC

If you happen to be working in Active Directory on an intranet, here are some tips:

(Windows Server 2012)

Running anything that talks to AD on a web server requires a bunch of changes and patience. Since when running on a web server vs. local IIS/IIS Express it runs in the AppPool's identity so, you have to set it up to impersonate whoever hits the site.

How to get the current logged-in user in an active directory when your ASP.NET MVC application is running on a web server inside the network:

// Find currently logged in user
UserPrincipal adUser = null;
using (HostingEnvironment.Impersonate())
{
    var userContext = System.Web.HttpContext.Current.User.Identity;
    PrincipalContext ctx = new PrincipalContext(ContextType.Domain, ConfigurationManager.AppSettings["AllowedDomain"], null,
                                                ContextOptions.Negotiate | ContextOptions.SecureSocketLayer);
    adUser = UserPrincipal.FindByIdentity(ctx, userContext.Name);
}
//Then work with 'adUser' from here...

You must wrap any calls having to do with 'active directory context' in the following so it's acting as the hosting environment to get the AD information:

using (HostingEnvironment.Impersonate()){ ... }

You must also have impersonate set to true in your web.config:

<system.web>
    <identity impersonate="true" />

You must have Windows authentication on in web.config:

<authentication mode="Windows" />

List vs tuple, when to use each?

I believe (and I am hardly well-versed in Python) that the main difference is that a tuple is immutable (it can't be changed in place after assignment) and a list is mutable (you can append, change, subtract, etc).

So, I tend to make my tuples things that shouldn't change after assignment and my lists things that can.

maven "cannot find symbol" message unhelpful

I had the same problem. The reason was that I had two JAR files were not added through the Maven dependency, so when I ran mvn compile, the console display the error error:

Symbol cannot be found,Class...".

To fix it:

  1. Remove the JAR files from build path
  2. Add to build path
  3. Run mvn compile

int value under 10 convert to string two digit number

The accepted answer is good and fast:

i.ToString("00")

or

i.ToString("000")

If you need more complexity, String.Format is worth a try:

var str1 = "";
var str2 = "";
for (int i = 1; i < 100; i++)
{
    str1 = String.Format("{0:00}", i);
    str2 = String.Format("{0:000}", i);
}

For the i = 10 case:

str1: "10"
str2: "010"

I use this, for example, to clear the text on particular Label Controls on my form by name:

private void EmptyLabelArray()
{
    var fmt = "Label_Row{0:00}_Col{0:00}";
    for (var rowIndex = 0; rowIndex < 100; rowIndex++)
    {
        for (var colIndex = 0; colIndex < 100; colIndex++)
        {
            var lblName = String.Format(fmt, rowIndex, colIndex);
            foreach (var ctrl in this.Controls)
            {
                var lbl = ctrl as Label;
                if ((lbl != null) && (lbl.Name == lblName))
                {
                    lbl.Text = null;
                }
            }
        }
    }
}

Getting value of HTML Checkbox from onclick/onchange events

Use this

<input type="checkbox" onclick="onClickHandler()" id="box" />

<script>
function onClickHandler(){
    var chk=document.getElementById("box").value;

    //use this value

}
</script>

How can I insert into a BLOB column from an insert statement in sqldeveloper?

Yes, it's possible, e.g. using the implicit conversion from RAW to BLOB:

insert into blob_fun values(1, hextoraw('453d7a34'));

453d7a34 is a string of hexadecimal values, which is first explicitly converted to the RAW data type and then inserted into the BLOB column. The result is a BLOB value of 4 bytes.

How to rollback just one step using rake db:migrate

Best way is running Particular migration again by using down or up(in rails 4. It's change)

rails db:migrate:up VERSION=timestamp

Now how you get the timestamp. Go to this path

/db/migrate

Identify migration file you want to revert.pick the timestamp from that file name.

Write to CSV file and export it?

A comment about Will's answer, you might want to replace HttpContext.Current.Response.End(); with HttpContext.Current.ApplicationInstance.CompleteRequest(); The reason is that Response.End() throws a System.Threading.ThreadAbortException. It aborts a thread. If you have an exception logger, it will be littered with ThreadAbortExceptions, which in this case is expected behavior.

Intuitively, sending a CSV file to the browser should not raise an exception.

See here for more Is Response.End() considered harmful?

Can dplyr package be used for conditional mutating?

case_when is now a pretty clean implementation of the SQL-style case when:

structure(list(a = c(1, 3, 4, 6, 3, 2, 5, 1), b = c(1, 3, 4, 
2, 6, 7, 2, 6), c = c(6, 3, 6, 5, 3, 6, 5, 3), d = c(6, 2, 4, 
5, 3, 7, 2, 6), e = c(1, 2, 4, 5, 6, 7, 6, 3), f = c(2, 3, 4, 
2, 2, 7, 5, 2)), .Names = c("a", "b", "c", "d", "e", "f"), row.names = c(NA, 
8L), class = "data.frame") -> df


df %>% 
    mutate( g = case_when(
                a == 2 | a == 5 | a == 7 | (a == 1 & b == 4 )     ~   2,
                a == 0 | a == 1 | a == 4 |  a == 3 | c == 4       ~   3
))

Using dplyr 0.7.4

The manual: http://dplyr.tidyverse.org/reference/case_when.html

'No JUnit tests found' in Eclipse

Right Click on Project > Properties > Java Build Path > Libraries > select classpath -> add Library -> Junit -> select junit version -> finish -> applay

Restore a deleted file in the Visual Studio Code Recycle Bin

I know the OP says Recycle Bin. What I do though is recreate the file, especially if it's a single file. And when in the file, I just press CMD+Z (I'm on a Mac) and I get my file back.

  1. Recreate the file in the same directory from where it was deleted.
  2. CMD+Z inside of the newly created file.

What are good examples of genetic algorithms/genetic programming solutions?

I developed a multithreaded swing based simulation of robot navigation through a set of randomized grid terrain of food sources and mines and developed a genetic algorithm based strategy of exploring the optimization of robotic behavior and survival of fittest genes for a robotic chromosome. This was done using charting and mapping of each iteration cycle.

Since, then I have developed even more game behavior. An example application I built recently for myself was a genetic algorithm for solving the traveling sales man problem in route finding in UK taking into account start and goal states as well as one/multiple connection points, delays, cancellations, construction works, rush hour, public strikes, consideration between fastest vs cheapest routes. Then providing a balanced recommendation for the route to take on a given day.

Generally, my strategy is to use POJO based representaton of genes then I apply specific interface implementations for selection, mutation, crossover strategies, and the criteria point. My fitness function then basically becomes a quite complex based on the strategy and criteria I need to apply as a heuristic measure.

I have also looked into applying genetic algorithm into automated testing within code using systematic mutation cycles where the algorithm understands the logic and tries to ascertain a bug report with recommendations for code fixes. Basically, a way to optimize my code and provide recommendations for improvement as well as a way of automating the discovery of new programmatic code. I have also tried to apply genetic algorithms to music production amongst other applications.

Generally, I find evolutionary strategies like most metaheuristic/global optimization strategies, they are slow to learn at first but start to pick up as the solutions become closer and closer to goal state and as long as your fitness function and heuristics are well aligned to produce that convergence within your search space.

Blue and Purple Default links, how to remove?

You need to override the color:

a { color:red } /* Globally */

/* Each state */

a:visited { text-decoration: none; color:red; }
a:hover { text-decoration: none; color:blue; }
a:focus { text-decoration: none; color:yellow; }
a:hover, a:active { text-decoration: none; color:black }

How can I determine the character encoding of an excel file?

For Excel 2010 it should be UTF-8. Instruction by MS :
http://msdn.microsoft.com/en-us/library/bb507946:

"The basic document structure of a SpreadsheetML document consists of the Sheets and Sheet elements, which reference the worksheets in the Workbook. A separate XML file is created for each Worksheet. For example, the SpreadsheetML for a workbook that has two worksheets name MySheet1 and MySheet2 is located in the Workbook.xml file and is shown in the following code example.

<?xml version="1.0" encoding="UTF-8" standalone="yes" ?> 
<workbook xmlns=http://schemas.openxmlformats.org/spreadsheetml/2006/main xmlns:r="http://schemas.openxmlformats.org/officeDocument/2006/relationships">
    <sheets>
        <sheet name="MySheet1" sheetId="1" r:id="rId1" /> 
        <sheet name="MySheet2" sheetId="2" r:id="rId2" /> 
    </sheets>
</workbook>

The worksheet XML files contain one or more block level elements such as SheetData. sheetData represents the cell table and contains one or more Row elements. A row contains one or more Cell elements. Each cell contains a CellValue element that represents the value of the cell. For example, the SpreadsheetML for the first worksheet in a workbook, that only has the value 100 in cell A1, is located in the Sheet1.xml file and is shown in the following code example.

<?xml version="1.0" encoding="UTF-8" ?> 
<worksheet xmlns="http://schemas.openxmlformats.org/spreadsheetml/2006/main">
    <sheetData>
        <row r="1">
            <c r="A1">
                <v>100</v> 
            </c>
        </row>
    </sheetData>
</worksheet>

"

Detection of cell encodings:

https://metacpan.org/pod/Spreadsheet::ParseExcel::Cell

http://forums.asp.net/t/1608228.aspx/1

How to SUM two fields within an SQL query

Due to my reputation points being less than 50 I could not comment on or vote for E Coder's answer above. This is the best way to do it so you don't have to use the group by as I had a similar issue.
By doing SUM((coalesce(VALUE1 ,0)) + (coalesce(VALUE2 ,0))) as Total this will get you the number you want but also rid you of any error for not performing a Group By. This was my query and gave me a total count and total amount for the each dealer and then gave me a subtotal for Quality and Risky dealer loans.

SELECT 
    DISTINCT STEP1.DEALER_NBR
    ,COUNT(*) AS DLR_TOT_CNT
    ,SUM((COALESCE(DLR_QLTY,0))+(COALESCE(DLR_RISKY,0))) AS DLR_TOT_AMT
    ,COUNT(STEP1.DLR_QLTY) AS DLR_QLTY_CNT
    ,SUM(STEP1.DLR_QLTY) AS DLR_QLTY_AMT
    ,COUNT(STEP1.DLR_RISKY) AS DLR_RISKY_CNT
    ,SUM(STEP1.DLR_RISKY) AS DLR_RISKY_AMT
    FROM STEP1
    WHERE DLR_QLTY IS NOT NULL OR DLR_RISKY IS NOT NULL
        GROUP BY STEP1.DEALER_NBR

Bootstrap carousel resizing image

add this to your css:

.carousel-inner > .item > img, .carousel-inner > .item > a > img {
    width: 100%;
}

How do I remove the first characters of a specific column in a table?

Stuff(someColumn, 1, 4, '')

This says, starting with the first 1 character position, replace 4 characters with nothing ''

Can I mask an input text in a bat file?

another alternative is my EditV32 (x86) or EditV64 (x64) command-line tools. For example:

editv32 -m -p "Password: " PWD

-m means "masked input" and -p is the prompt. The user's input is stored in the PWD environment variable. You can get it here:

https://westmesatech.com/?page_id=19

MySQL query to select events between start/end date

If I understood correctly you are trying to use a single query, i think you can just merge your date search toghter in WHERE clauses

SELECT id 
FROM events 
WHERE start BETWEEN '2013-06-13' AND '2013-07-22' 
AND end BETWEEN '2013-06-13' AND '2013-07-22'

or even more simply you can just use both column to set search time filter

SELECT id 
FROM events 
WHERE start >= '2013-07-22' AND end <= '2013-06-13'

SQL: How do I SELECT only the rows with a unique value on certain column?

Sorry you're not using PostgreSQL...

SELECT DISTINCT ON contract, activity * FROM thetable ORDER BY contract, activity

http://www.postgresql.org/docs/8.3/static/sql-select.html#SQL-DISTINCT

Oh wait. You only want values with exactly one...

SELECT contract, activity, count() FROM thetable GROUP BY contract, activity HAVING count() = 1

Passing Variable through JavaScript from one html page to another page

There are two pages: Pageone.html :

<script>
var hello = "hi"
location.replace("http://example.com/PageTwo.html?" + hi + "");
</script>

PageTwo.html :

<script>
var link = window.location.href;
link = link.replace("http://example.com/PageTwo.html?","");
document.write("The variable contained this content:" + link + "");
</script>

Hope it helps!

How to fix Uncaught InvalidValueError: setPosition: not a LatLng or LatLngLiteral: in property lat: not a number?

What about casting to Number in Javascript using the Unary (+) Operator

lat: 12.23
lat: +12.23

Depends on the use case. Here is a fullchart what works and what not: parseInt vs unary plus - when to use which

How to specify the private SSH-key to use when executing shell command on Git?

If SSH port number is not 22(default), add Port xx in ~/.ssh/config

In my case (synology),

Host my_synology
    Hostname xxxx.synology.me
    IdentityFile ~/.ssh/id_rsa_xxxx
    User myname
    Port xx

Then clone using Host title in config. ("my_synology". to avoid @chopstik 's "*")

git clone my_synology:path/to/repo.git

PHP - Check if the page run on Mobile or Desktop browser

I used Robert Lee`s answer and it works great! Just writing down the complete function i'm using:

function isMobileDevice(){
    $aMobileUA = array(
        '/iphone/i' => 'iPhone', 
        '/ipod/i' => 'iPod', 
        '/ipad/i' => 'iPad', 
        '/android/i' => 'Android', 
        '/blackberry/i' => 'BlackBerry', 
        '/webos/i' => 'Mobile'
    );

    //Return true if Mobile User Agent is detected
    foreach($aMobileUA as $sMobileKey => $sMobileOS){
        if(preg_match($sMobileKey, $_SERVER['HTTP_USER_AGENT'])){
            return true;
        }
    }
    //Otherwise return false..  
    return false;
}

How to dump a table to console?

Most pure lua print table functions I've seen have a problem with deep recursion and tend to cause a stack overflow when going too deep. This print table function that I've written does not have this problem. It should also be capable of handling really large tables due to the way it handles concatenation. In my personal usage of this function, it outputted 63k lines to file in about a second.

The output also keeps lua syntax and the script can easily be modified for simple persistent storage by writing the output to file if modified to allow only number, boolean, string and table data types to be formatted.

function print_table(node)
    local cache, stack, output = {},{},{}
    local depth = 1
    local output_str = "{\n"

    while true do
        local size = 0
        for k,v in pairs(node) do
            size = size + 1
        end

        local cur_index = 1
        for k,v in pairs(node) do
            if (cache[node] == nil) or (cur_index >= cache[node]) then

                if (string.find(output_str,"}",output_str:len())) then
                    output_str = output_str .. ",\n"
                elseif not (string.find(output_str,"\n",output_str:len())) then
                    output_str = output_str .. "\n"
                end

                -- This is necessary for working with HUGE tables otherwise we run out of memory using concat on huge strings
                table.insert(output,output_str)
                output_str = ""

                local key
                if (type(k) == "number" or type(k) == "boolean") then
                    key = "["..tostring(k).."]"
                else
                    key = "['"..tostring(k).."']"
                end

                if (type(v) == "number" or type(v) == "boolean") then
                    output_str = output_str .. string.rep('\t',depth) .. key .. " = "..tostring(v)
                elseif (type(v) == "table") then
                    output_str = output_str .. string.rep('\t',depth) .. key .. " = {\n"
                    table.insert(stack,node)
                    table.insert(stack,v)
                    cache[node] = cur_index+1
                    break
                else
                    output_str = output_str .. string.rep('\t',depth) .. key .. " = '"..tostring(v).."'"
                end

                if (cur_index == size) then
                    output_str = output_str .. "\n" .. string.rep('\t',depth-1) .. "}"
                else
                    output_str = output_str .. ","
                end
            else
                -- close the table
                if (cur_index == size) then
                    output_str = output_str .. "\n" .. string.rep('\t',depth-1) .. "}"
                end
            end

            cur_index = cur_index + 1
        end

        if (size == 0) then
            output_str = output_str .. "\n" .. string.rep('\t',depth-1) .. "}"
        end

        if (#stack > 0) then
            node = stack[#stack]
            stack[#stack] = nil
            depth = cache[node] == nil and depth + 1 or depth - 1
        else
            break
        end
    end

    -- This is necessary for working with HUGE tables otherwise we run out of memory using concat on huge strings
    table.insert(output,output_str)
    output_str = table.concat(output)

    print(output_str)
end

Here is an example:

local t = {
    ["abe"] = {1,2,3,4,5},
    "string1",
    50,
    ["depth1"] = { ["depth2"] = { ["depth3"] = { ["depth4"] = { ["depth5"] = { ["depth6"] = { ["depth7"]= { ["depth8"] = { ["depth9"] = { ["depth10"] = {1000}, 900}, 800},700},600},500}, 400 }, 300}, 200}, 100},
    ["ted"] = {true,false,"some text"},
    "string2",
    [function() return end] = function() return end,
    75
}

print_table(t)

Output:

{
    [1] = 'string1',
    [2] = 50,
    [3] = 'string2',
    [4] = 75,
    ['abe'] = {
        [1] = 1,
        [2] = 2,
        [3] = 3,
        [4] = 4,
        [5] = 5
    },
    ['function: 06472B70'] = 'function: 06472A98',
    ['depth1'] = {
        [1] = 100,
        ['depth2'] = {
            [1] = 200,
            ['depth3'] = {
                [1] = 300,
                ['depth4'] = {
                    [1] = 400,
                    ['depth5'] = {
                        [1] = 500,
                        ['depth6'] = {
                            [1] = 600,
                            ['depth7'] = {
                                [1] = 700,
                                ['depth8'] = {
                                    [1] = 800,
                                    ['depth9'] = {
                                        [1] = 900,
                                        ['depth10'] = {
                                            [1] = 1000
                                        }
                                    }
                                }
                            }
                        }
                    }
                }
            }
        }
    },
    ['ted'] = {
        [1] = true,
        [2] = false,
        [3] = 'some text'
    }
}

java.lang.ClassNotFoundException: org.springframework.web.servlet.DispatcherServlet

It may be useful for someone, so I'll post it here. I was missing this dependency on my pom.xml

<dependency>
    <groupId>org.springframework.boot</groupId>
    <artifactId>spring-boot-starter-web</artifactId>
    <exclusions>
        <exclusion>
            <groupId>org.springframework.boot</groupId>
            <artifactId>spring-boot-starter-tomcat</artifactId>
        </exclusion>
    </exclusions>
</dependency>

MySQL Orderby a number, Nulls last

Something like

SELECT * FROM tablename where visible=1 ORDER BY COALESCE(position, 999999999) ASC, id DESC

Replace 999999999 with what ever the max value for the field is

Installing Python 2.7 on Windows 8

GUI Option:

  1. Open System Properties

    a. Type it in the Start Menu

    b. Use the keyboard shortcut Win+Pause)

    c. From Windows Explorer address bar go to

    %windir%\System32\SystemPropertiesProtection.exe

    d. Write SystemPropertiesProtection in run window and press Enter

  2. Switch to the Advanced tab

  3. Click Environment Variables
  4. Select PATH in the System variables section
  5. Click Edit
  6. Add python's path to the end of the list (the paths are separated by semicolons).

For example:

C:\Windows;C:\Windows\System32;C:\Python27

Command Line Option:

  1. Run Command Prompt as administrator
  2. Check existing paths under PATH variable (the paths are separated by semicolons). If your python folder already listed then no need to add again. Default python folder is C:\Python27

    C:\Windows\system32>path or C:\Windows\system32>echo %PATH%

  3. Append python path using setx command. The /M option sets the variable at SYSTEM scope.

The default behavior is to set it for the USER.

C:\Windows\system32>setx /M PATH "%PATH%;C:\Python27"

How do I pass multiple parameters into a function in PowerShell?

I don't see it mentioned here, but splatting your arguments is a useful alternative and becomes especially useful if you are building out the arguments to a command dynamically (as opposed to using Invoke-Expression). You can splat with arrays for positional arguments and hashtables for named arguments. Here are some examples:

Splat With Arrays (Positional Arguments)

Test-Connection with Positional Arguments

Test-Connection www.google.com localhost

With Array Splatting

$argumentArray = 'www.google.com', 'localhost'
Test-Connection @argumentArray

Note that when splatting, we reference the splatted variable with an @ instead of a $. It is the same when using a Hashtable to splat as well.

Splat With Hashtable (Named Arguments)

Test-Connection with Named Arguments

Test-Connection -ComputerName www.google.com -Source localhost

With Hashtable Splatting

$argumentHash = @{
  ComputerName = 'www.google.com'
  Source = 'localhost'
}
Test-Connection @argumentHash

Splat Positional and Named Arguments Simultaneously

Test-Connection With Both Positional and Named Arguments

Test-Connection www.google.com localhost -Count 1

Splatting Array and Hashtables Together

$argumentHash = @{
  Count = 1
}
$argumentArray = 'www.google.com', 'localhost'
Test-Connection @argumentHash @argumentArray

Declare and Initialize String Array in VBA

Using

Dim myarray As Variant

works but

Dim myarray As String

doesn't so I sitck to Variant

VBA ADODB excel - read data from Recordset

I am surprised that the connection string works for you, because it is missing a semi-colon. Set is only used with objects, so you would not say Set strNaam.

Set cn = CreateObject("ADODB.Connection")
With cn
 .Provider = "Microsoft.Jet.OLEDB.4.0"
  .ConnectionString = "Data Source=D:\test.xls " & _
  ";Extended Properties=""Excel 8.0;HDR=Yes;"""
.Open
End With
strQuery = "SELECT * FROM [Sheet1$E36:E38]"
Set rs = cn.Execute(strQuery)
Do While Not rs.EOF
  For i = 0 To rs.Fields.Count - 1
    Debug.Print rs.Fields(i).Name, rs.Fields(i).Value
    strNaam = rs.Fields(0).Value
  Next
  rs.MoveNext
Loop
rs.Close

There are other ways, depending on what you want to do, such as GetString (GetString Method Description).

Check whether a path is valid

private bool IsValidPath(string path)
{
    Regex driveCheck = new Regex(@"^[a-zA-Z]:\\$");

    if (string.IsNullOrWhiteSpace(path) || path.Length < 3)
    {
        return false;
    }

    if (!driveCheck.IsMatch(path.Substring(0, 3)))
    {
        return false;
    }

    var x1 = (path.Substring(3, path.Length - 3));
    string strTheseAreInvalidFileNameChars = new string(Path.GetInvalidPathChars());
    strTheseAreInvalidFileNameChars += @":?*";
    Regex containsABadCharacter = new Regex("[" + Regex.Escape(strTheseAreInvalidFileNameChars) + "]");

    if (containsABadCharacter.IsMatch(path.Substring(3, path.Length - 3)))
    {
        return false;
    }

    var driveLetterWithColonAndSlash = Path.GetPathRoot(path);

    if (!DriveInfo.GetDrives().Any(x => x.Name == driveLetterWithColonAndSlash))
    {
        return false;
    }

    return true;
}

Qt: How do I handle the event of the user pressing the 'X' (close) button?

You can attach a SLOT to the

void aboutToQuit();

signal of your QApplication. This signal should be raised just before app closes.

$(document).on("click"... not working?

An old post, but I love to share as I have the same case but I finally knew the problem :

Problem is : We make a function to work with specified an HTML element, but the HTML element related to this function is not yet created (because the element was dynamically generated). To make it works, we should make the function at the same time we create the element. Element first than make function related to it.

Simply word, a function will only works to the element that created before it (him). Any elements that created dynamically means after him.

But please inspect this sample that did not heed the above case :

<div class="btn-list" id="selected-country"></div>

Dynamically appended :

<button class="btn-map" data-country="'+country+'">'+ country+' </button>

This function is working good by clicking the button :

$(document).ready(function() {    
        $('#selected-country').on('click','.btn-map', function(){ 
        var datacountry = $(this).data('country'); console.log(datacountry);
    });
})

or you can use body like :

$('body').on('click','.btn-map', function(){ 
            var datacountry = $(this).data('country'); console.log(datacountry);
        });

compare to this that not working :

$(document).ready(function() {     
$('.btn-map').on("click", function() { 
        var datacountry = $(this).data('country'); alert(datacountry);
    });
});

hope it will help

iPhone: Setting Navigation Bar Title

For all your Swift-ers out there, this worked perfectly for me. It's notably one of the shorter ways to accomplish setting the title, as well:

override public func prepareForSegue(segue: UIStoryboardSegue, sender: AnyObject?) {
  if segue.identifier == "presentLineItem" {
    print("Setting Title")
    var vc = segue.destinationViewController as! LineItemsTableViewController
    vc.navigationItem.title = "Line Item"
  }
}

split string only on first instance of specified character

This solution worked for me

var str = "good_luck_buddy";
var index = str.indexOf('_');
var arr = [str.slice(0, index), str.slice(index + 1)];

//arr[0] = "good"
//arr[1] = "luck_buddy"

OR

var str = "good_luck_buddy";
var index = str.indexOf('_');
var [first, second] = [str.slice(0, index), str.slice(index + 1)];

//first = "good"
//second = "luck_buddy"

Python check if list items are integers?

You can use exceptional handling as str.digit will only work for integers and can fail for something like this too:

>>> str.isdigit(' 1')
False

Using a generator function:

def solve(lis):                                        
    for x in lis:
        try:
            yield float(x)
        except ValueError:    
            pass

>>> mylist = ['1','orange','2','3','4','apple', '1.5', '2.6']
>>> list(solve(mylist))                                    
[1.0, 2.0, 3.0, 4.0, 1.5, 2.6]   #returns converted values

or may be you wanted this:

def solve(lis):
    for x in lis:
        try:
            float(x)
            return True
        except:
            return False
...         
>>> mylist = ['1','orange','2','3','4','apple', '1.5', '2.6']
>>> [x for x in mylist if solve(x)]
['1', '2', '3', '4', '1.5', '2.6']

or using ast.literal_eval, this will work for all types of numbers:

>>> from ast import literal_eval
>>> def solve(lis):
    for x in lis:
        try:
            literal_eval(x)
            return True
        except ValueError:   
             return False
...         
>>> mylist=['1','orange','2','3','4','apple', '1.5', '2.6', '1+0j']
>>> [x for x in mylist if solve(x)]                               
['1', '2', '3', '4', '1.5', '2.6', '1+0j']

Number of occurrences of a character in a string

Why use regex for that. String implements IEnumerable<char>, so you can just use LINQ.

test.Count(c => c == '&')

IIS7 Permissions Overview - ApplicationPoolIdentity

Part A: Configuring your Application Pool

Suppose the Application Pool is named 'MyPool' Go to 'Advanced Settings' of the Application Pool from the IIS Manager

  1. Scroll down to 'Identity'. Trying to edit the value will bring up a dialog box. Select 'Built-In account' and under it, select 'ApplicationPoolIdentity'.

  2. A few lines below 'Identity', you should find 'Load User Profile'. This value should be set to 'True'.

Part B: Configuring your website

  1. Website Name: SiteName (just an example)
  2. Physical Path: C:\Whatever (just an example)
  3. Connect as... : Application User (pass-through authentication) (The above settings can be found in 'Basic Settings' of the site in the IIS Manager)
  4. After configuring the basic settings, look for the 'Authentication' configuration under 'IIS' in the main console of the site. Open it. You should see an option for 'Anonymous Authentication'. Make sure it is enabled. Then right click and 'Edit...' it. Select 'Application Pool Identity'.

Part C: Configuring your folder

The folder in question is C:\Whatever

  1. Go to Properties - Sharing - Advanced Sharing - Permissions, and tick 'Share this folder'
  2. In the same dialog box, you will find a button 'Permissions'. Click it.
  3. A new dialog box will open. Click 'Add'.
  4. A new dialog box 'Select Users or Groups' will open. Under 'From this location' make sure the name is the same as your local host computer. Then, under 'Enter the object names', type 'IIS AppPool\MyPool' and click 'Check Names' and then 'Ok'
  5. Give full sharing permissions for 'MyPool' user. Apply it and close the folder properties
  6. Open folder properties again. This time, go to Security - Advanced - Permission, and click Add. There will be an option 'Select a Principal' at the top, or some other option to choose a user. Click it.
  7. The 'Select Users or Groups' dialog box will open again. Repeat step 4.
  8. Give all or as many permissions you need to the 'MyPool' user.
  9. Check 'Replace all child object permissions..." and Apply and close.

You should now be able to use the browse the website

What is the T-SQL syntax to connect to another SQL Server?

on my C drive I first create a txt file to create a new table. You can use what ever you want in this text file

in this case the text file is called "Bedrijf.txt"

the content:

Print 'START(A) create table'

GO 1

If not EXISTS
(
    SELECT *
    FROM INFORMATION_SCHEMA.TABLES
    WHERE TABLE_NAME = 'Bedrijf'
)
BEGIN
CREATE TABLE [dbo].[Bedrijf] (
[IDBedrijf] [varchar] (38)   NOT NULL ,
[logo] [varbinary] (max) NULL ,
[VolledigeHandelsnaam] [varchar] (100)  NULL 
) ON [PRIMARY]

save it

then I create an other txt file with the name "Bedrijf.bat" and the extension bat. It's content:

OSQL.EXE  -U Username -P Password -S IPaddress -i C:Bedrijf.txt -o C:Bedrijf.out -d myDatabaseName

save it and from explorer double click to execute

The results will be saved in a txt file on your C drive with the name "Bedrijf.out"

it shows

1> 2> 3> START(A) create table

if all goes well

That's it

PowerShell script to return members of multiple security groups

This is cleaner and will put in a csv.

Import-Module ActiveDirectory

$Groups = (Get-AdGroup -filter * | Where {$_.name -like "**"} | select name -expandproperty name)


$Table = @()

$Record = [ordered]@{
"Group Name" = ""
"Name" = ""
"Username" = ""
}



Foreach ($Group in $Groups)
{

$Arrayofmembers = Get-ADGroupMember -identity $Group | select name,samaccountname

foreach ($Member in $Arrayofmembers)
{
$Record."Group Name" = $Group
$Record."Name" = $Member.name
$Record."UserName" = $Member.samaccountname
$objRecord = New-Object PSObject -property $Record
$Table += $objrecord

}

}

$Table | export-csv "C:\temp\SecurityGroups.csv" -NoTypeInformation

Git pull till a particular commit

If you merge a commit into your branch, you should get all the history between.

Observe:

$ git init ./
Initialized empty Git repository in /Users/dfarrell/git/demo/.git/
$ echo 'a' > letter
$ git add letter
$ git commit -m 'Initial Letter'
[master (root-commit) 6e59e76] Initial Letter
 1 file changed, 1 insertion(+)
 create mode 100644 letter
$ echo 'b' >> letter
$ git add letter && git commit -m 'Adding letter'
[master 7126e6d] Adding letter
 1 file changed, 1 insertion(+)
$ echo 'c' >> letter; git add letter && git commit -m 'Adding letter'
[master f2458be] Adding letter
 1 file changed, 1 insertion(+)
$ echo 'd' >> letter; git add letter && git commit -m 'Adding letter'
[master 7f77979] Adding letter
 1 file changed, 1 insertion(+)
$ echo 'e' >> letter; git add letter && git commit -m 'Adding letter'
[master 790eade] Adding letter
 1 file changed, 1 insertion(+)
$ git log
commit 790eade367b0d8ab8146596cd717c25fd895302a
Author: Dan Farrell 
Date:   Thu Jul 16 14:21:26 2015 -0500

    Adding letter

commit 7f77979efd17f277b4be695c559c1383d2fc2f27
Author: Dan Farrell 
Date:   Thu Jul 16 14:21:24 2015 -0500

    Adding letter

commit f2458bea7780bf09fe643095dbae95cf97357ccc
Author: Dan Farrell 
Date:   Thu Jul 16 14:21:19 2015 -0500

    Adding letter

commit 7126e6dcb9c28ac60cb86ae40fb358350d0c5fad
Author: Dan Farrell 
Date:   Thu Jul 16 14:20:52 2015 -0500

    Adding letter

commit 6e59e7650314112fb80097d7d3803c964b3656f0
Author: Dan Farrell 
Date:   Thu Jul 16 14:20:33 2015 -0500

    Initial Letter
$ git checkout 6e59e7650314112fb80097d7d3803c964b3656f
$ git checkout 7126e6dcb9c28ac60cb86ae40fb358350d0c5fad
Note: checking out '7126e6dcb9c28ac60cb86ae40fb358350d0c5fad'.

You are in 'detached HEAD' state. You can look around, make experimental
changes and commit them, and you can discard any commits you make in this
state without impacting any branches by performing another checkout.

If you want to create a new branch to retain commits you create, you may
do so (now or later) by using -b with the checkout command again. Example:

  git checkout -b new_branch_name

HEAD is now at 7126e6d... Adding letter
$ git checkout -b B 7126e6dcb9c28ac60cb86ae40fb358350d0c5fad
Switched to a new branch 'B'
$ git pull 790eade367b0d8ab8146596cd717c25fd895302a
fatal: '790eade367b0d8ab8146596cd717c25fd895302a' does not appear to be a git repository
fatal: Could not read from remote repository.

Please make sure you have the correct access rights
and the repository exists.
$ git merge 7f77979efd17f277b4be695c559c1383d2fc2f27
Updating 7126e6d..7f77979
Fast-forward
 letter | 2 ++
 1 file changed, 2 insertions(+)
$ cat letter
a
b
c
d

Google Chrome default opening position and size

First, close all instances of Google Chrome. There should be no instances of chrome.exe running in the Windows Task Manager. Then

  • Go to %LOCALAPPDATA%\Google\Chrome\User Data\Default\.
  • Open the file "Preferences" in a text editor like Notepad.
  • First, resave the file to something like "Preference - Old" without any extension (i.e. no .txt). This will serve as a backup, should something go wrong.
  • Look for a section called "browser." Inside that section, you should find a subsection called window_placement. Under window_placement you will see things like "bottom", "left", "right", etc. with numbers after them.

You will need to play around with these numbers to get your desired window size and placement. When finished, save this file with the name "Preferences" again with no extension. This will overwrite the existing Preferences file. Open Chrome and see how you did. If you're not satisfied with the size and placement, close Chrome and change the numbers in the Preferences file until you get what you want.

Warning :-Presenting view controllers on detached view controllers is discouraged

Lots of reasons for this warning. Mine is because I have a segue connected from a ViewController to another that will be presented modally. But, the ViewController I am presenting from is being dynamically generated by a PageViewController. Which is why it is detached in the Storyboard. My app isn't going to crash because of it; but I would like to silence the warning.

console.writeline and System.out.println

There's no Console.writeline in Java. Its in .NET.

Console and standard out are not same. If you read the Javadoc page you mentioned, you will see that an application can have access to a console only if it is invoked from the command line and the output is not redirected like this

java -jar MyApp.jar > MyApp.log

Other such cases are covered in SimonJ's answer, though he missed out on the point that there's no Console.writeline.

Unable to get provider com.google.firebase.provider.FirebaseInitProvider

I've got this error just on devices with API lower that 21. In my case, I have had to work with a project where multiDexEnabled option in build.gradle was already set to true. I checked dex file from APK and referenced methods number was less than 64k, so project doesn't need to be a multidex one, therefore I set multiDexEnabled to false. This solution worked for me.

Local and global temporary tables in SQL Server

It is worth mentioning that there is also: database scoped global temporary tables(currently supported only by Azure SQL Database).

Global temporary tables for SQL Server (initiated with ## table name) are stored in tempdb and shared among all users’ sessions across the whole SQL Server instance.

Azure SQL Database supports global temporary tables that are also stored in tempdb and scoped to the database level. This means that global temporary tables are shared for all users’ sessions within the same Azure SQL Database. User sessions from other databases cannot access global temporary tables.

-- Session A creates a global temp table ##test in Azure SQL Database testdb1
-- and adds 1 row
CREATE TABLE ##test ( a int, b int);
INSERT INTO ##test values (1,1);

-- Session B connects to Azure SQL Database testdb1 
-- and can access table ##test created by session A
SELECT * FROM ##test
---Results
1,1

-- Session C connects to another database in Azure SQL Database testdb2 
-- and wants to access ##test created in testdb1.
-- This select fails due to the database scope for the global temp tables 
SELECT * FROM ##test
---Results
Msg 208, Level 16, State 0, Line 1
Invalid object name '##test'

ALTER DATABASE SCOPED CONFIGURATION

GLOBAL_TEMPORARY_TABLE_AUTODROP = { ON | OFF }

APPLIES TO: Azure SQL Database (feature is in public preview)

Allows setting the auto-drop functionality for global temporary tables. The default is ON, which means that the global temporary tables are automatically dropped when not in use by any session. When set to OFF, global temporary tables need to be explicitly dropped using a DROP TABLE statement or will be automatically dropped on server restart.

With Azure SQL Database single databases and elastic pools, this option can be set in the individual user databases of the SQL Database server. In SQL Server and Azure SQL Database managed instance, this option is set in TempDB and the setting of the individual user databases has no effect.

How do I remove all non alphanumeric characters from a string except dash?

Here's an extension method using @ata answer as inspiration.

"hello-world123, 456".MakeAlphaNumeric(new char[]{'-'});// yields "hello-world123456"

or if you require additional characters other than hyphen...

"hello-world123, 456!?".MakeAlphaNumeric(new char[]{'-','!'});// yields "hello-world123456!"


public static class StringExtensions
{   
    public static string MakeAlphaNumeric(this string input, params char[] exceptions)
    {
        var charArray = input.ToCharArray();
        var alphaNumeric = Array.FindAll<char>(charArray, (c => char.IsLetterOrDigit(c)|| exceptions?.Contains(c) == true));
        return new string(alphaNumeric);
    }
}

How to detect window.print() finish

Given that you wish to wait for the print dialog to go away I would use focus binding on the window.

print();

var handler = function(){
    //unbind task();
    $(window).unbind("focus",handler);
}

$(window).bind("focus",handler);

By putting in the unbind in the handler function we prevent the focus event staying bond to the window.

Spring Boot application as a Service

Here is a script that deploys an executable jar as a systemd service.

It creates a user for the service and the .service file, and place the jar file under /var, and makes some basic lock down of privileges.

#!/bin/bash

# Argument: The jar file to deploy
APPSRCPATH=$1

# Argument: application name, no spaces please, used as folder name under /var
APPNAME=$2

# Argument: the user to use when running the application, may exist, created if not exists
APPUSER=$3

# Help text
USAGE="
Usage: sudo $0 <jar-file> <app-name> <runtime-user>
If an app with the name <app-name> already exist, it is stopped and deleted.
If the <runtime-user> does not already exist, it is created.
"

# Check that we are root
if [ ! "root" = "$(whoami)" ]; then
    echo "Must be root. Please use e.g. sudo"
    echo "$USAGE"
    exit
fi

# Check arguments
if [ "$#" -ne 3 -o ${#APPSRCPATH} = 0 -o ${#APPNAME} = 0 -o ${#APPUSER} = 0 ]; then
    echo "Incorrect number of parameters."
    echo "$USAGE"
    exit
fi

if [ ! -f $APPSRCPATH ]; then
    echo "Can't find jar file $APPSRCPATH"
    echo "$USAGE"
    exit
fi

# Infered values
APPFILENAME=$(basename $APPSRCPATH)
APPFOLDER=/var/javaapps/$APPNAME
APPDESTPATH=$APPFOLDER/$APPFILENAME

# Stop the service if it already exist and is running
systemctl stop $APPNAME >/dev/null 2>&1

# Create the app folder, deleting any previous content
rm -fr $APPFOLDER
mkdir -p $APPFOLDER

# Create the user if it does not exist
if id "$APPUSER" >/dev/null 2>&1; then
    echo "Using existing user $APPUSER"
else
    adduser --disabled-password --gecos "" $APPUSER
    echo "Created user $APPUSER"
fi

# Place app in app folder, setting owner and rights
cp $APPSRCPATH $APPDESTPATH
chown $APPUSER $APPDESTPATH
chmod 500 $APPDESTPATH
echo "Added or updated the $APPDESTPATH file"

# Create the .service file used by systemd
echo "
[Unit]
Description=$APPNAME
After=syslog.target
[Service]
User=$APPUSER
ExecStart=/usr/bin/java -jar $APPDESTPATH
SuccessExitStatus=143
[Install]
WantedBy=multi-user.target
" > /etc/systemd/system/$APPNAME.service
echo "Created the /etc/systemd/system/$APPNAME.service file"

# Reload the daemon
systemctl daemon-reload

# Start the deployed app
systemctl start $APPNAME
systemctl status $APPNAME

Example: enter image description here

Defining TypeScript callback type

To go one step further, you could declare a type pointer to a function signature like:

interface myCallbackType { (myArgument: string): void }

and use it like this:

public myCallback : myCallbackType;

iptables block access to port 8000 except from IP address

This question should be on Server Fault. Nevertheless, the following should do the trick, assuming you're talking about TCP and the IP you want to allow is 1.2.3.4:

iptables -A INPUT -p tcp --dport 8000 -s 1.2.3.4 -j ACCEPT
iptables -A INPUT -p tcp --dport 8000 -j DROP

How to set the opacity/alpha of a UIImage?

If you're experimenting with Metal rendering & you're extracting the CGImage generated by imageByApplyingAlpha in the first reply, you may end up with a Metal rendering that's larger than you expect. While experimenting with Metal, you may want to change one line of code in imageByApplyingAlpha:

    UIGraphicsBeginImageContextWithOptions (self.size, NO, 1.0f);
//  UIGraphicsBeginImageContextWithOptions (self.size, NO, 0.0f);

If you're using a device with a scale factor of 3.0, like the iPhone 11 Pro Max, the 0.0 scale factor shown above will give you an CGImage that's three times larger than you're expecting. Changing the scale factor to 1.0 should avoid any scaling.

Hopefully, this reply will save beginners a lot of aggravation.

curl error 18 - transfer closed with outstanding read data remaining

I've solved this error by this way.

$ch = curl_init ();
curl_setopt ( $ch, CURLOPT_URL, 'http://www.someurl/' );
curl_setopt ( $ch, CURLOPT_TIMEOUT, 30);
ob_start();
$response = curl_exec ( $ch );
$data = ob_get_clean();
if(curl_getinfo($ch, CURLINFO_HTTP_CODE) == 200 ) success;

Error still occurs, but I can handle response data in variable.

moment.js - UTC gives wrong date

By default, MomentJS parses in local time. If only a date string (with no time) is provided, the time defaults to midnight.

In your code, you create a local date and then convert it to the UTC timezone (in fact, it makes the moment instance switch to UTC mode), so when it is formatted, it is shifted (depending on your local time) forward or backwards.

If the local timezone is UTC+N (N being a positive number), and you parse a date-only string, you will get the previous date.

Here are some examples to illustrate it (my local time offset is UTC+3 during DST):

>>> moment('07-18-2013', 'MM-DD-YYYY').utc().format("YYYY-MM-DD HH:mm")
"2013-07-17 21:00"
>>> moment('07-18-2013 12:00', 'MM-DD-YYYY HH:mm').utc().format("YYYY-MM-DD HH:mm")
"2013-07-18 09:00"
>>> Date()
"Thu Jul 25 2013 14:28:45 GMT+0300 (Jerusalem Daylight Time)"

If you want the date-time string interpreted as UTC, you should be explicit about it:

>>> moment(new Date('07-18-2013 UTC')).utc().format("YYYY-MM-DD HH:mm")
"2013-07-18 00:00"

or, as Matt Johnson mentions in his answer, you can (and probably should) parse it as a UTC date in the first place using moment.utc() and include the format string as a second argument to prevent ambiguity.

>>> moment.utc('07-18-2013', 'MM-DD-YYYY').format("YYYY-MM-DD HH:mm")
"2013-07-18 00:00"

To go the other way around and convert a UTC date to a local date, you can use the local() method, as follows:

>>> moment.utc('07-18-2013', 'MM-DD-YYYY').local().format("YYYY-MM-DD HH:mm")
"2013-07-18 03:00"

Where can I find Android's default icons?

you can use

android.R.drawable.xxx

(use autocomplete to see whats in there)

Or download the stuff from http://developer.android.com/design/downloads/index.html

How to check if two arrays are equal with JavaScript?

With JavaScript version 1.6 it's as easy as this:

Array.prototype.equals = function( array ) {
  return this.length == array.length && 
         this.every( function(this_i,i) { return this_i == array[i] } )  
  }

For example, [].equals([]) gives true, while [1,2,3].equals( [1,3,2] ) yields false.

Initialize value of 'var' in C# to null

var variables still have a type - and the compiler error message says this type must be established during the declaration.

The specific request (assigning an initial null value) can be done, but I don't recommend it. It doesn't provide an advantage here (as the type must still be specified) and it could be viewed as making the code less readable:

var x = (String)null;

Which is still "type inferred" and equivalent to:

String x = null;

The compiler will not accept var x = null because it doesn't associate the null with any type - not even Object. Using the above approach, var x = (Object)null would "work" although it is of questionable usefulness.

Generally, when I can't use var's type inference correctly then

  1. I am at a place where it's best to declare the variable explicitly; or
  2. I should rewrite the code such that a valid value (with an established type) is assigned during the declaration.

The second approach can be done by moving code into methods or functions.

Visual Studio 2010 always thinks project is out of date, but nothing has changed

I'm on Visual Studio 2013 and just updated to the Windows 10 May 2019 update and compiling suddenly had to be redone every time, regardless of changes. Tried renaming the pch to ProjectName instead of TargetName, looked for missing files with the detailed log and that Python script, but in the end it was my time was not synced with MS's servers (by like milliseconds).

What resolved this for me was

  • "Adjust date and time" in the control panel
  • "Sync Now"

Now my projects don't need to be recompiled for no reason.

Async await in linq select

I prefer this as an extension method:

public static async Task<IEnumerable<T>> WhenAll<T>(this IEnumerable<Task<T>> tasks)
{
    return await Task.WhenAll(tasks);
}

So that it is usable with method chaining:

var inputs = await events
  .Select(async ev => await ProcessEventAsync(ev))
  .WhenAll()

How to tag docker image with docker-compose

It seems the docs/tool have been updated and you can now add the image tag to your script. This was successful for me.

Example:

version: '2'
services:

  baggins.api.rest:
    image: my.image.name:rc2
    build:
      context: ../..
      dockerfile: app/Docker/Dockerfile.release
    ports:
      ...

https://docs.docker.com/compose/compose-file/#build

CGRectMake, CGPointMake, CGSizeMake, CGRectZero, CGPointZero is unavailable in Swift

If you want to use them as in swift 2, you can use these funcs:

For CGRectMake:

func CGRectMake(_ x: CGFloat, _ y: CGFloat, _ width: CGFloat, _ height: CGFloat) -> CGRect {
    return CGRect(x: x, y: y, width: width, height: height)
}

For CGPointMake:

func CGPointMake(_ x: CGFloat, _ y: CGFloat) -> CGPoint {
    return CGPoint(x: x, y: y)
}

For CGSizeMake:

func CGSizeMake(_ width: CGFloat, _ height: CGFloat) -> CGSize {
    return CGSize(width: width, height: height)
}

Just put them outside any class and it should work. (Works for me at least)

What is "loose coupling?" Please provide examples

Many integrated products (especially by Apple) such as iPods, iPads are a good example of tight coupling: once the battery dies you might as well buy a new device because the battery is soldered fixed and won't come loose, thus making replacing very expensive. A loosely coupled player would allow effortlessly changing the battery.

The same goes for software development: it is generally (much) better to have loosely coupled code to facilitate extension and replacement (and to make individual parts easier to understand). But, very rarely, under special circumstances tight coupling can be advantageous because the tight integration of several modules allows for better optimisation.

How to get the cookie value in asp.net website

HttpCookie cook = new HttpCookie("testcook");
cook = Request.Cookies["CookName"];
if (cook != null)
{
    lbl_cookie_value.Text = cook.Value;
}
else
{
    lbl_cookie_value.Text = "Empty value";
}

Reference Click here

Is there a /dev/null on Windows?

If you need to perform in Microsoft Windows the equivalent of a symlink to /dev/null in Linux you would open and administrator's cmd and type:

For files:

mklink c:\path\to\file.ext NUL:

Or, for directories:

mklink /D c:\path\to\dir NUL:

This will keep the file/direcotry always at 0 byte, and still return success to every write attempt.

How to add image to canvas

here is the sample code to draw image on canvas-

$("#selectedImage").change(function(e) {

var URL = window.URL;
var url = URL.createObjectURL(e.target.files[0]);
img.src = url;

img.onload = function() {
    var canvas = document.getElementById("myCanvas");
    var ctx = canvas.getContext("2d");        

    ctx.clearRect(0, 0, canvas.width, canvas.height);
    ctx.drawImage(img, 0, 0, 500, 500);
}});

In the above code selectedImage is an input control which can be used to browse image on system. For more details of sample code to draw image on canvas while maintaining the aspect ratio:

http://newapputil.blogspot.in/2016/09/show-image-on-canvas-html5.html

How to change Git log date formats

Use Bash and the date command to convert from an ISO-like format to the one you want. I wanted an org-mode date format (and list item), so I did this:

echo + [$(date -d "$(git log --pretty=format:%ai -1)" +"%Y-%m-%d %a %H:%M")] \
    $(git log --pretty=format:"%h %s" --abbrev=12 -1)

And the result is for example:

+ [2015-09-13 Sun 22:44] 2b0ad02e6cec Merge pull request #72 from 3b/bug-1474631

How to show text in combobox when no item selected?

Use the insert method of the combobox to insert the "Please select item" in to 0 index,

comboBox1.Items.Insert(0, "Please select any value");

and add all the items to the combobox after the first index. In the form load set

comboBox1.SelectedIndex = 0;

EDIT:

In form load write the text in to the comboBox1.Text by hardcoding

comboBox1.Text = "Please, select any value";

and in the TextChanged event of the comboBox1 write the following code

 private void comboBox1_TextChanged(object sender, EventArgs e)
        {
            if (comboBox1.SelectedIndex < 0)
            {
                comboBox1.Text = "Please, select any value";
            }
            else
            {
                comboBox1.Text = comboBox1.SelectedText;
            }
        }

Print number of keys in Redis

dbsize() returns the total number of keys.

You can quickly estimate the number of keys matching a given pattern by sampling keys at random, then checking what fraction of them matches the pattern.

Example in python; counting all keys starting with prefix_:

import redis
r = redis.StrictRedis(host = 'localhost', port=6379)
iter=1000
print 'Approximately', r.dbsize() * float(sum([r.randomkey().startswith('prefix_') for i in xrange(iter)])) / iter

Even iter=100 gives a decent estimate in my case, yet is very fast, compared to keys prefix_.

An improvement is to sample 1000 keys on every request, but keep the total count, so that after two requests you'll divide by 2000, after three requests you'll divide by 3000. Thus, if your application is interested in the total number of matching keys fairly often, then every time it will get closer and closer to the true value.

I forgot the password I entered during postgres installation

The file .pgpass in a user's home directory or the file referenced by PGPASSFILE can contain passwords to be used if the connection requires a password (and no password has been specified otherwise). On Microsoft Windows the file is named %APPDATA%\postgresql\pgpass.conf (where %APPDATA% refers to the Application Data subdirectory in the user's profile).

This file should contain lines of the following format:

hostname:port:database:username:password

(You can add a reminder comment to the file by copying the line above and preceding it with #.) Each of the first four fields can be a literal value, or *, which matches anything. The password field from the first line that matches the current connection parameters will be used. (Therefore, put more-specific entries first when you are using wildcards.) If an entry needs to contain : or \, escape this character with . A host name of localhost matches both TCP (host name localhost) and Unix domain socket (pghost empty or the default socket directory) connections coming from the local machine. In a standby server, a database name of replication matches streaming replication connections made to the master server. The database field is of limited usefulness because users have the same password for all databases in the same cluster.

On Unix systems, the permissions on .pgpass must disallow any access to world or group; achieve this by the command chmod 0600 ~/.pgpass. If the permissions are less strict than this, the file will be ignored. On Microsoft Windows, it is assumed that the file is stored in a directory that is secure, so no special permissions check is made.

How to connect HTML Divs with Lines?

Definitely possible with any number of libraries and/or HTML5 technologies. You could possible hack something together in pure CSS by using something like the border-bottom property, but it would probably be horribly hacky.

If you're serious about this, you should take a look at a JS library for canvas drawing or SVG. For example, something like http://www.graphjs.org/ or http://jsdraw2dx.jsfiction.com/

Start systemd service after specific service?

In the .service file under the [Unit] section:

[Unit]
Description=My Website
After=syslog.target network.target mongodb.service

The important part is the mongodb.service

The manpage describes it however due to formatting it's not as clear on first sight

systemd.unit - well formatted

systemd.unit - not so well formatted

Redirect on Ajax Jquery Call

For ExpressJs router:

router.post('/login', async(req, res) => {
    return res.send({redirect: '/yoururl'});
})

Client-side:

    success: function (response) {
        if (response.redirect) {
            window.location = response.redirect
        }
    },

How to get JQuery.trigger('click'); to initiate a mouse click

You just need to put a small timeout event before doing .click() like this :

setTimeout(function(){ $('#btn').click()}, 100);

How to force div to appear below not next to another?

Float the #list and #similar the right and add clear: right; to #similar

Like so:

#map { float:left; width:700px; height:500px; }
#list { float:right; width:200px; background:#eee; list-style:none; padding:0; }
#similar { float:right; width:200px; background:#000; clear:right; } 


<div id="map"></div>        
<ul id="list"></ul>
<div id="similar">this text should be below, not next to ul.</div>

You might need a wrapper(div) around all of them though that's the same width of the left and right element.

ffmpeg - Converting MOV files to MP4

The command to just stream it to a new container (mp4) needed by some applications like Adobe Premiere Pro without encoding (fast) is:

ffmpeg -i input.mov -qscale 0 output.mp4

Alternative as mentioned in the comments, which re-encodes with best quaility (-qscale 0):

ffmpeg -i input.mov -q:v 0 output.mp4

How to simplify a null-safe compareTo() implementation?

Another Apache ObjectUtils example. Able to sort other types of objects.

@Override
public int compare(Object o1, Object o2) {
    String s1 = ObjectUtils.toString(o1);
    String s2 = ObjectUtils.toString(o2);
    return s1.toLowerCase().compareTo(s2.toLowerCase());
}

Best practices with STDIN in Ruby?

Following are some things I found in my collection of obscure Ruby.

So, in Ruby, a simple no-bells implementation of the Unix command cat would be:

#!/usr/bin/env ruby
puts ARGF.read

ARGF is your friend when it comes to input; it is a virtual file that gets all input from named files or all from STDIN.

ARGF.each_with_index do |line, idx|
    print ARGF.filename, ":", idx, ";", line
end

# print all the lines in every file passed via command line that contains login
ARGF.each do |line|
    puts line if line =~ /login/
end

Thank goodness we didn’t get the diamond operator in Ruby, but we did get ARGF as a replacement. Though obscure, it actually turns out to be useful. Consider this program, which prepends copyright headers in-place (thanks to another Perlism, -i) to every file mentioned on the command-line:

#!/usr/bin/env ruby -i

Header = DATA.read

ARGF.each_line do |e|
  puts Header if ARGF.pos - e.length == 0
  puts e
end

__END__
#--
# Copyright (C) 2007 Fancypants, Inc.
#++

Credit to:

How to generate an entity-relationship (ER) diagram using Oracle SQL Developer

Create a diagram for existing database schema or its subset as follows:

  1. Click File ? Data Modeler ? Import ? Data Dictionary.
  2. Select a DB connection (add one if none).
  3. Click Next.
  4. Check one or more schema names.
  5. Click Next.
  6. Check one or more objects to import.
  7. Click Next.
  8. Click Finish.

The ERD is displayed.

Export the diagram as follows:

  1. Click File ? Data Modeler ? Print Diagram ? To Image File.
  2. Browse to and select the export file location.
  3. Click Save.

The diagram is exported. To export in a vector format, use To PDF File, instead. This allows for simplified editing using Inkscape (or other vector image editor).

These instructions may work for SQL Developer 3.2.09.23 to 4.1.3.20.

Allow multiple roles to access controller action

One possible simplification would be to subclass AuthorizeAttribute:

public class RolesAttribute : AuthorizeAttribute
{
    public RolesAttribute(params string[] roles)
    {
        Roles = String.Join(",", roles);
    }
}

Usage:

[Roles("members", "admin")]

Semantically it is the same as Jim Schmehil's answer.

How to send email from Terminal?

For SMTP hosts and Gmail I like to use Swaks -> https://easyengine.io/tutorials/mail/swaks-smtp-test-tool/

On a Mac:

  1. brew install swaks
  2. swaks --to [email protected] --server smtp.example.com

Using `window.location.hash.includes` throws “Object doesn't support property or method 'includes'” in IE11

I'm using ReactJs and used import 'core-js/es6/string'; at the start of index.js to solve my problem.

I'm also using import 'react-app-polyfill/ie11'; to support running React in IE11.

react-app-polyfill

This package includes polyfills for various browsers. It includes minimum requirements and commonly used language features used by Create React App projects.

https://github.com/facebook/create-react-app/blob/master/packages/react-app-polyfill/README.md

Android Fastboot devices not returning device

TLDR - In addition to the previous responses. There might be a problem with the version of the fastboot command. Try to download the newest one via Android SDK Manager instead of default one available in the OS repository.

There is one more thing you can do to fix this issue. I had the similar problem when trying to flash Nexus Player. All the adb commands we working fine while in normal boot mode. However, after switching to fastboot mode I wasn't able to execute fastboot commands. My device was not visible in the output of the fastboot devices command. I've set the right rules in /etc/udev/rules.d/11-android.rules file. The lsusb command showed that the device has been pluged in.

The soultion was quite simple. I've downloaded the the fastboot via Android Studio's SDK Manager instead of using the default one available in Ubuntu packages.

All you need is sdkmanager. Download the Android SDK Platform Tools and replace the default /usr/bin/fastboot with the new one.

JavaScript Chart Library

Fusion charts has a new javascript/jquery library that looks promising.

GDB: Listing all mapped memory regions for a crashed process

You can also use info files to list all the sections of all the binaries loaded in process binary.

How to hide .php extension in .htaccess

1) Are you sure mod_rewrite module is enabled? Check phpinfo()

2) Your above rule assumes the URL starts with "folder". Is this correct? Did you acutally want to have folder in the URL? This would match a URL like:

/folder/thing -> /folder/thing.php

If you actually want

/thing -> /folder/thing.php

You need to drop the folder from the match expression.

I usually use this to route request to page without php (but yours should work which leads me to think that mod_rewrite may not be enabled):

RewriteRule ^([^/\.]+)/?$ $1.php  [L,QSA]

3) Assuming you are declaring your rules in an .htaccess file, does your installation allow for setting Options (AllowOverride) overrides in .htaccess files? Some shared hosts do not.

When the server finds an .htaccess file (as specified by AccessFileName) it needs to know which directives declared in that file can override earlier access information.

How to include a child object's child object in Entity Framework 5

If you include the library System.Data.Entity you can use an overload of the Include() method which takes a lambda expression instead of a string. You can then Select() over children with Linq expressions rather than string paths.

return DatabaseContext.Applications
     .Include(a => a.Children.Select(c => c.ChildRelationshipType));

OS X Terminal shortcut: Jump to beginning/end of line

As setup in the terminal using vi:

The Home button on a Macbook Pro keyboard: Fn + Left Arrow.

The End button on a Macbook Pro keyboard: Fn + Right Arrow.

Difference between two dates in Python

pd.date_range('2019-01-01', '2019-02-01').shape[0]

Best way to check if a Data Table has a null value in it

You can null/blank/space Etc value using LinQ Use Following Query

   var BlankValueRows = (from dr1 in Dt.AsEnumerable()
                                  where dr1["Columnname"].ToString() == ""
                                  || dr1["Columnname"].ToString() == ""
                                   || dr1["Columnname"].ToString() == ""
                                  select Columnname);

Here Replace Columnname with table column name and "" your search item in above code we looking null value.

socket.error: [Errno 10013] An attempt was made to access a socket in a way forbidden by its access permissions

I found a solution to solve this problem in Python.

go to c:\python27\ directory and rigtlcick python.exe and tab to compaitbility and select the admin privilege option and apply the changes. Now you issue the command it allows to create the socket connection.

How to customize a Spinner in Android

I have build a small demo project on this you could have a look to it Link to project

Implement Stack using Two Queues

import java.util.Deque;
import java.util.LinkedList;
import java.util.Queue;

public class ImplementationOfStackUsingTwoQueue {

    private static Deque<Integer> inboxQueue = new LinkedList<>();
    private static Queue<Integer> outboxQueue = new LinkedList<>();
    
    public void pushInStack(Integer val){
        inboxQueue.add(val);
    }
    
    public void popFromStack(){
        
        if(outboxQueue.isEmpty()){
            while(!inboxQueue.isEmpty()){
                outboxQueue.add(inboxQueue.pollLast());
            }
        }
    }
    
    public static void main(String[] args) {
        
        ImplementationOfStackUsingTwoQueue obj = new ImplementationOfStackUsingTwoQueue();
        
        obj.pushInStack(1);
        obj.pushInStack(2);
        obj.pushInStack(3);
        obj.pushInStack(4);
        obj.pushInStack(5);
        System.out.println("After pushing the values in Queue" + inboxQueue);
        
        obj.popFromStack();
        System.out.println("After popping the values from Queue" + outboxQueue);    
    }
}

Android splash screen image sizes to fit all devices

Using PNG is not such a good idea. Actually it's costly as far as performance is concerned. You can use drawable XML files, for example, Facebook's background.

This will help you to smooth and speed up your performance, and for the logo use .9 patch images.

How to sort a list of strings numerically?

Try this, it’ll sort the list in-place in descending order (there’s no need to specify a key in this case):

Process

listB = [24, 13, -15, -36, 8, 22, 48, 25, 46, -9]
listC = sorted(listB, reverse=True) # listB remains untouched
print listC

output:

 [48, 46, 25, 24, 22, 13, 8, -9, -15, -36]

reading a line from ifstream into a string variable

Use the std::getline() from <string>.

 istream & getline(istream & is,std::string& str)

So, for your case it would be:

std::getline(read,x);

using nth-child in tables tr td

Current css version still doesn't support selector find by content. But there is a way, by using css selector find by attribute, but you have to put some identifier on all of the <td> that have $ inside. Example: using nth-child in tables tr td

html

<tr>
    <td>&nbsp;</td>
    <td data-rel='$'>$</td>
    <td>&nbsp;</td>
</tr>

css

table tr td[data-rel='$'] {
    background-color: #333;
    color: white;
}

Please try these example.

_x000D_
_x000D_
table tr td[data-content='$'] {_x000D_
    background-color: #333;_x000D_
    color: white;_x000D_
}
_x000D_
<table border="1">_x000D_
    <tr>_x000D_
        <td>A</td>_x000D_
        <td data-content='$'>$</td>_x000D_
        <td>B</td>_x000D_
        <td data-content='$'>$</td>_x000D_
        <td>C</td>_x000D_
        <td data-content='$'>$</td>_x000D_
        <td>D</td>_x000D_
    </tr>_x000D_
</table>
_x000D_
_x000D_
_x000D_

Can an XSLT insert the current date?

XSLT 2

Date functions are available natively, such as:

<xsl:value-of  select="current-dateTime()"/>

There is also current-date() and current-time().

XSLT 1

Use the EXSLT date and times extension package.

  1. Download the date and times package from GitHub.
  2. Extract date.xsl to the location of your XSL files.
  3. Set the stylesheet header.
  4. Import date.xsl.

For example:

<xsl:stylesheet version="1.0" 
    xmlns:date="http://exslt.org/dates-and-times" 
    extension-element-prefixes="date"
    ...>

    <xsl:import href="date.xsl" />

    <xsl:template match="//root">
       <xsl:value-of select="date:date-time()"/>
    </xsl:template>
</xsl:stylesheet>

How to upload a file from Windows machine to Linux machine using command lines via PuTTy?

Use putty. Put install directory path in environment values (PATH), and restart your PC if required.

Open cmd (command prompt) and type

C:/> pscp "C:\Users/gsjha/Desktop/example.txt" user@host:/home/

It'll be copied to the system.

What is the difference between a symbolic link and a hard link?

IN this answer when i say a file i mean the location in memory

All the data that is saved is stored in memory using a data structure called inodes Every inode has a inodenumber.The inode number is used to access the inode.All the hard links to a file may have different names but share the same inode number.Since all the hard links have the same inodenumber(which inturn access the same inode),all of them point to the same physical memory.

A symbolic link is a special kind of file.Since it is also a file it will have a file name and an inode number.As said above the inode number acceses an inode which points to data.Now what makes a symbolic link special is that the inodenumbers in symbolic links access those inodes which point to "a path" to another file.More specifically the inode number in symbolic link acceses those inodes who point to another hard link.

when we are moving,copying,deleting a file in GUI we are playing with the hardlinks of the file not the physical memory.when we delete a file we are deleting the hardlink of the file. we are not wiping out the physical memory.If all the hardlinks to file are deleted then it will not be possible to access the data stored although it may still be present in memory

How can I make a link from a <td> table cell

Yes, that's possible, albeit not literally the <td>, but what's in it. The simple trick is, to make sure that the content extends to the borders of the cell (it won't include the borders itself though).

As already explained, this isn't semantically correct. An a element is an inline element and should not be used as block-level element. However, here's an example (but JavaScript plus a td:hover CSS style will be much neater) that works in most browsers:

<td>
  <a href="http://example.com">
    <div style="height:100%;width:100%">
      hello world
    </div>
  </a>
</td>

PS: it's actually neater to change a in a block-level element using CSS as explained in another solution in this thread. it won't work well in IE6 though, but that's no news ;)

Alternative (non-advised) solution

If your world is only Internet Explorer (rare, nowadays), you can violate the HTML standard and write this, it will work as expected, but will be highly frowned upon and be considered ill-advised (you haven't heard this from me). Any other browser than IE will not render the link, but will show the table correctly.

<table>
    <tr>
        <a href="http://example.com"><td  width="200">hello world</td></a>
    </tr>
</table>

Change URL without refresh the page

When you use a function ...

<p onclick="update_url('/en/step2');">Link</p>

<script>
function update_url(url) {
    history.pushState(null, null, url);
}
</script>

Credit card payment gateway in PHP?

There are more than a few gateways out there, but I am not aware of a reliable gateway that is free. Most gateways like PayPal will provide you APIs that will allow you to process credit cards, as well as do things like void, charge, or refund.

The other thing you need to worry about is the coming of PCI compliance which basically says if you are not compliant, you (or the company you work for) will be liable by your Merchant Bank and/or Card Vendor for not being compliant by July of 2010. This will impose large fines on you and possibly revoke the ability for you to process credit cards.

All that being said companies like PayPal have a PHP SDK:

https://cms.paypal.com/us/cgi-bin/?cmd=_render-content&content_ID=developer/library_download_sdks

Authorize.Net:

http://developer.authorize.net/samplecode/

Those are two of the more popular ones for the United States.

For PCI Info see:

https://www.pcisecuritystandards.org/

ASP.NET MVC ActionLink and post method

I have done the same issue using following code:

@using (Html.BeginForm("Delete", "Admin"))
{
       @Html.Hidden("ProductID", item.ProductID)
       <input type="submit" value="Delete" />
}

What is the best way to test for an empty string with jquery-out-of-the-box?

if (!a) {
  // is emtpy
}

To ignore white space for strings:

if (!a.trim()) {
    // is empty or whitespace
}

If you need legacy support (IE8-) for trim(), use $.trim or a polyfill.

SCCM 2012 application install "Failed" in client Software Center

The execmgr.log will show the commandline and ccmcache folder used for installation. Typically, required apps don't show on appenforce.log and some clients will have outdated appenforce or no ppenforce.log files. execmgr.log also shows required hidden uninstall actions as well.

You may want to save the blog link. I still reference it from time to time.

How do I create documentation with Pydoc?

Another thing that people may find useful...make sure to leave off ".py" from your module name. For example, if you are trying to generate documentation for 'original' in 'original.py':

yourcode_dir$ pydoc -w original.py
no Python documentation found for 'original.py'

yourcode_dir$ pydoc -w original
wrote original.html

How to Ignore "Duplicate Key" error in T-SQL (SQL Server)

Use IGNORE_DUP_KEY = OFF during primary key definition to ignore the duplicates while insert. for example

create table X( col1.....)

CONSTRAINT [pk_X] PRIMARY KEY CLUSTERED 
(
)WITH (PAD_INDEX  = OFF, STATISTICS_NORECOMPUTE  = OFF, IGNORE_DUP_KEY = OFF, ALLOW_ROW_LOCKS  = ON, ALLOW_PAGE_LOCKS  = ON, FILLFACTOR = 70) ON [PRIMARY]
) ON [PRIMARY]

Jenkins - how to build a specific branch

enter image description here

There will be an option in configure under Build Triggers

Check the GitHub Branches

A hook will be created and then you can build any branch you like from Jenkins when you select github Branches enter image description here

Hope it helps :)

How to change credentials for SVN repository in Eclipse?

I figured out the method I have written below, and it works perfectly for me

in eclipse, follow these simple steps: File -> Import -> type svn -> project from svn -> next -> Create a new repository location -> Next - > You would see "Secure Storage" below the Authentication Menu. Click on this button "Secure Storage" . This is the place where we can delete all the password caches. Now once you are here, follow these:

  1. Under password tab, click "Clear passwords"
  2. Under "contents" tab, go to SVN, select each sub options, and click on "Delete"
  3. Restart eclipse

All the stored passwords are now gone, and you can use your new svn password to sync

MVVM Passing EventArgs As Command Parameter

To add to what joshb has stated already - this works just fine for me. Make sure to add references to Microsoft.Expression.Interactions.dll and System.Windows.Interactivity.dll and in your xaml do:

    xmlns:i="http://schemas.microsoft.com/expression/2010/interactivity"

I ended up using something like this for my needs. This shows that you can also pass a custom parameter:

<i:Interaction.Triggers>
            <i:EventTrigger EventName="SelectionChanged">

                <i:InvokeCommandAction Command="{Binding Path=DataContext.RowSelectedItem, RelativeSource={RelativeSource AncestorType={x:Type Window}}}" 
                                       CommandParameter="{Binding Path=SelectedItem, RelativeSource={RelativeSource Mode=FindAncestor, AncestorType=DataGrid}}" />
            </i:EventTrigger>
</i:Interaction.Triggers>

How to draw a line in android

You can draw multiple straight lines on view using Finger paint example which is in Developer android. example link

Just comment: mPath.quadTo(mX, mY, (x + mX)/2, (y + mY)/2); You will be able to draw straight lines.

import android.app.Activity;
import android.content.Context;
import android.graphics.Bitmap;
import android.graphics.Canvas;
import android.graphics.Color;
import android.graphics.Paint;
import android.graphics.Path;
import android.graphics.Point;
import android.os.Bundle;
import android.view.MotionEvent;
import android.view.View;
import android.view.View.OnTouchListener;
import android.widget.ImageView;

public class JoinPointsActivity extends Activity  {
    /** Called when the activity is first created. */
    Paint mPaint;
    float Mx1,My1;
    float x,y;
    @Override

    public void onCreate(Bundle savedInstanceState) {
        super.onCreate(savedInstanceState);
       // setContentView(R.layout.main);
        MyView view1 =new MyView(this);
        view1.setBackgroundResource(R.drawable.image_0031_layer_1);
        setContentView(view1);


        mPaint = new Paint();
        mPaint.setAntiAlias(true);
        mPaint.setDither(true);
        mPaint.setColor(0xFFFF0000);
        mPaint.setStyle(Paint.Style.STROKE);
        mPaint.setStrokeJoin(Paint.Join.ROUND);
       // mPaint.setStrokeCap(Paint.Cap.ROUND);
        mPaint.setStrokeWidth(10);

    }

    public class MyView extends View {

        private static final float MINP = 0.25f;
        private static final float MAXP = 0.75f;

      private Bitmap  mBitmap;
        private Canvas  mCanvas;
        private Path    mPath;
       private Paint   mBitmapPaint;

        public MyView(Context c) {
            super(c);

            mPath = new Path();
          mBitmapPaint = new Paint(Paint.DITHER_FLAG);
        }

        @Override
       protected void onSizeChanged(int w, int h, int oldw, int oldh) {
            super.onSizeChanged(w, h, oldw, oldh);
            mBitmap = Bitmap.createBitmap(w, h, Bitmap.Config.ARGB_8888);
            mCanvas = new Canvas(mBitmap);
        }

        @Override
        protected void onDraw(Canvas canvas) {
            canvas.drawColor(0xFFAAAAAA);
           // canvas.drawLine(mX, mY, Mx1, My1, mPaint);
           // canvas.drawLine(mX, mY, x, y, mPaint);
            canvas.drawBitmap(mBitmap, 0, 0, mBitmapPaint);
            canvas.drawPath(mPath, mPaint);

        }

        private float mX, mY;
        private static final float TOUCH_TOLERANCE = 4;

        private void touch_start(float x, float y) {
            mPath.reset();
            mPath.moveTo(x, y);
            mX = x;
            mY = y;
        }
        private void touch_move(float x, float y) {
            float dx = Math.abs(x - mX);
            float dy = Math.abs(y - mY);
            if (dx >= TOUCH_TOLERANCE || dy >= TOUCH_TOLERANCE) {
               // mPath.quadTo(mX, mY, (x + mX)/2, (y + mY)/2);
                mX = x;
                mY = y;
            }
        }
        private void touch_up() {
            mPath.lineTo(mX, mY);
            // commit the path to our offscreen
            mCanvas.drawPath(mPath, mPaint);
            // kill this so we don't double draw
            mPath.reset();
        }

        @Override
        public boolean onTouchEvent(MotionEvent event) {
            float x = event.getX();
            float y = event.getY();

            switch (event.getAction()) {
                case MotionEvent.ACTION_DOWN:
                    touch_start(x, y);
                    invalidate();
                    break;
                case MotionEvent.ACTION_MOVE:
                    touch_move(x, y);
                    invalidate();
                    break;
                case MotionEvent.ACTION_UP:
                    touch_up();
               //   Mx1=(int) event.getX();
                 //  My1= (int) event.getY();
                   invalidate();
                    break;
            }
            return true;
        }
    }

}

Stack Memory vs Heap Memory

In C++ the stack memory is where local variables get stored/constructed. The stack is also used to hold parameters passed to functions.

The stack is very much like the std::stack class: you push parameters onto it and then call a function. The function then knows that the parameters it expects can be found on the end of the stack. Likewise, the function can push locals onto the stack and pop them off it before returning from the function. (caveat - compiler optimizations and calling conventions all mean things aren't this simple)

The stack is really best understood from a low level and I'd recommend Art of Assembly - Passing Parameters on the Stack. Rarely, if ever, would you consider any sort of manual stack manipulation from C++.

Generally speaking, the stack is preferred as it is usually in the CPU cache, so operations involving objects stored on it tend to be faster. However the stack is a limited resource, and shouldn't be used for anything large. Running out of stack memory is called a Stack buffer overflow. It's a serious thing to encounter, but you really shouldn't come across one unless you have a crazy recursive function or something similar.

Heap memory is much as rskar says. In general, C++ objects allocated with new, or blocks of memory allocated with the likes of malloc end up on the heap. Heap memory almost always must be manually freed, though you should really use a smart pointer class or similar to avoid needing to remember to do so. Running out of heap memory can (will?) result in a std::bad_alloc.

Mockito - difference between doReturn() and when()

Continuing this answer, There is another difference that if you want your method to return different values for example when it is first time called, second time called etc then you can pass values so for example...

PowerMockito.doReturn(false, false, true).when(SomeClass.class, "SomeMethod", Matchers.any(SomeClass.class));

So it will return false when the method is called in same test case and then it will return false again and lastly true.

How can I check if a string is null or empty in PowerShell?

Note that the "if ($str)" and "IsNullOrEmpty" tests don't work comparably in all instances: an assignment of $str=0 produces false for both, and depending on intended program semantics, this could yield a surprise.

Entity framework linq query Include() multiple children entities

Might be it will help someone, 4 level and 2 child's on each level

Library.Include(a => a.Library.Select(b => b.Library.Select(c => c.Library)))
            .Include(d=>d.Book.)
            .Include(g => g.Library.Select(h=>g.Book))
            .Include(j => j.Library.Select(k => k.Library.Select(l=>l.Book)))

Default fetch type for one-to-one, many-to-one and one-to-many in Hibernate

For Single valued associations, i.e.-One-to-One and Many-to-One:-
Default Lazy=proxy
Proxy lazy loading:- This implies a proxy object of your associated entity is loaded. This means that only the id connecting the two entities is loaded for the proxy object of associated entity.
Eg: A and B are two entities with Many to one association. ie: There may be multiple A's for every B. Every object of A will contain a reference of B.
`

public class A{
    int aid;
    //some other A parameters;
    B b;
}
public class B{
    int bid;
     //some other B parameters;
}

`
The relation A will contain columns(aid,bid,...other columns of entity A).
The relation B will contain columns(bid,...other columns of entity B)

Proxy implies when A is fetched, only id is fetched for B and stored into a proxy object of B which contains only id. Proxy object of B is a an object of a proxy-class which is a subclass of B with only minimal fields. Since bid is already part of relation A, it is not necessary to fire a query to get bid from the relation B. Other attributes of entity B are lazily loaded only when a field other than bid is accessed.

For Collections, i.e.-Many-to-Many and One-to-Many:-
Default Lazy=true


Please also note that the fetch strategy(select,join etc) can override lazy. ie: If lazy='true' and fetch='join', fetching of A will also fetch B or Bs(In case of collections). You can get the reason if you think about it.
Default fetch for single valued association is "join".
Default fetch for collections is "select". Please verify the last two lines. I have deduced that logically.

How to drop column with constraint?

You can also drop the column and its constraint(s) in a single statement rather than individually.

CREATE TABLE #T
  (
     Col1 INT CONSTRAINT UQ UNIQUE CONSTRAINT CK CHECK (Col1 > 5),
     Col2 INT
  )

ALTER TABLE #T DROP CONSTRAINT UQ , 
                    CONSTRAINT CK, 
                    COLUMN Col1


DROP TABLE #T 

Some dynamic SQL that will look up the names of dependent check constraints and default constraints and drop them along with the column is below

(but not other possible column dependencies such as foreign keys, unique and primary key constraints, computed columns, indexes)

CREATE TABLE [dbo].[TestTable]
(
A INT DEFAULT '1' CHECK (A=1),
B INT,
CHECK (A > B)
)

GO

DECLARE @TwoPartTableNameQuoted nvarchar(500) = '[dbo].[TestTable]',
        @ColumnNameUnQuoted sysname = 'A',
        @DynSQL NVARCHAR(MAX);

SELECT @DynSQL =
     'ALTER TABLE ' + @TwoPartTableNameQuoted + ' DROP' + 
      ISNULL(' CONSTRAINT ' + QUOTENAME(OBJECT_NAME(c.default_object_id)) + ',','') + 
      ISNULL(check_constraints,'') + 
      '  COLUMN ' + QUOTENAME(@ColumnNameUnQuoted)
FROM   sys.columns c
       CROSS APPLY (SELECT ' CONSTRAINT ' + QUOTENAME(OBJECT_NAME(referencing_id)) + ','
                    FROM   sys.sql_expression_dependencies
                    WHERE  referenced_id = c.object_id
                           AND referenced_minor_id = c.column_id
                           AND OBJECTPROPERTYEX(referencing_id, 'BaseType') = 'C'
                    FOR XML PATH('')) ck(check_constraints)
WHERE  c.object_id = object_id(@TwoPartTableNameQuoted)
       AND c.name = @ColumnNameUnQuoted;

PRINT @DynSQL;
EXEC (@DynSQL); 

omp parallel vs. omp parallel for

I don't think there is any difference, one is a shortcut for the other. Although your exact implementation might deal with them differently.

The combined parallel worksharing constructs are a shortcut for specifying a parallel construct containing one worksharing construct and no other statements. Permitted clauses are the union of the clauses allowed for the parallel and worksharing contructs.

Taken from http://www.openmp.org/mp-documents/OpenMP3.0-SummarySpec.pdf

The specs for OpenMP are here:

https://openmp.org/specifications/

How do I convert a Swift Array to a String?

A separator can be a bad idea for some languages like Hebrew or Japanese. Try this:

// Array of Strings
let array: [String] = ["red", "green", "blue"]
let arrayAsString: String = array.description
let stringAsData = arrayAsString.data(using: String.Encoding.utf16)
let arrayBack: [String] = try! JSONDecoder().decode([String].self, from: stringAsData!)

For other data types respectively:

// Set of Doubles
let set: Set<Double> = [1, 2.0, 3]
let setAsString: String = set.description
let setStringAsData = setAsString.data(using: String.Encoding.utf16)
let setBack: Set<Double> = try! JSONDecoder().decode(Set<Double>.self, from: setStringAsData!)

Using import fs from 'fs'

ES6 modules support in Node.js is fairly recent; even in the bleeding-edge versions, it is still experimental. With Node.js 10, you can start Node.js with the --experimental-modules flag, and it will likely work.

To import on older Node.js versions - or standard Node.js 10 - use CommonJS syntax:

const fs = require('fs');

Dropping a connected user from an Oracle 10g database schema

Find existing sessions to DB using this query:

SELECT s.inst_id,
       s.sid,
       s.serial#,
       p.spid,
       s.username,
       s.program
FROM   gv$session s
       JOIN gv$process p ON p.addr = s.paddr AND p.inst_id = s.inst_id
WHERE  s.type != 'BACKGROUND';

you'll see something like below. Oracle Sessions

Then, run below query with values extracted from above results.

ALTER SYSTEM KILL SESSION '<put above s.sid here>,<put above s.serial# here>';

Ex: ALTER SYSTEM KILL SESSION '93,943';

"Couldn't read dependencies" error with npm

Verify user account, you are working on. If any system user has no permissions for installation packages, npm particulary also is showing this message.

Version of Apache installed on a Debian machine

Surely typing /usr/sbin/apache2 -v into the shell is the best and quickest way to go, by the way here's another option, just in case there's PHP too in your server and you're interested into gathering Apache version (and much more infos) in quick programmatical steps.

Just make an info.php file in your Apache web root folder (or whatever you like) and write these inside it:

<?php
    phpinfo();
?>

Now go to yoursite.com/info.php (or localhost/info.php for local machines).

You'll see your Apache version in PHP Variables section, here's an example:

phpinfo() example dump page

Also, please notice that these steps obviously apply to any web server with PHP integration, so it's not limited to Apache and once created that page could be handy while developing (just don't forget to remove it on production environments!!)

Batch Extract path and filename from a variable

if you want infos from the actual running batchfile, try this :

@echo off
set myNameFull=%0
echo myNameFull     %myNameFull%
set myNameShort=%~n0
echo myNameShort    %myNameShort%
set myNameLong=%~nx0
echo myNameLong     %myNameLong%
set myPath=%~dp0
echo myPath         %myPath%
set myLogfileWpath=%myPath%%myNameShort%.log
echo myLogfileWpath %myLogfileWpath%

more samples? C:> HELP CALL

%0 = parameter 0 = batchfile %1 = parameter 1 - 1st par. passed to batchfile... so you can try that stuff (e.g. "~dp") between 1st (e.g. "%") and last (e.g. "1") also for parameters

What is the method for converting radians to degrees?

Here is some code which extends Object with rad(deg), deg(rad) and also two more useful functions: getAngle(point1,point2) and getDistance(point1,point2) where a point needs to have a x and y property.

Object.prototype.rad = (deg) => Math.PI/180 * deg;
Object.prototype.deg = (rad) => 180/Math.PI * rad;
Object.prototype.getAngle = (point1, point2) => Math.atan2(point1.y - point2.y, point1.x - point2.x);
Object.prototype.getDistance = (point1, point2) => Math.sqrt(Math.pow(point1.x-point2.x, 2) + Math.pow(point1.y-point2.y, 2));

CSS root directory

For example your directory is like this:

Desktop >
        ProjectFolder >
                      index.html
                      css >
                          style.css
                      images >
                             img.png

You are at your style.css and you want to use img.png as a background-image, use this:

url("../images/img.png")

Works for me!

Detecting iOS orientation change instantly

For my case handling UIDeviceOrientationDidChangeNotification was not good solution as it is called more frequent and UIDeviceOrientation is not always equal to UIInterfaceOrientation because of (FaceDown, FaceUp).

I handle it using UIApplicationDidChangeStatusBarOrientationNotification:

//To add the notification
[[NSNotificationCenter defaultCenter] addObserver:self selector:@selector(didChangeOrientation:)

//to remove the
[[NSNotificationCenter defaultCenter]removeObserver:self name:UIDeviceOrientationDidChangeNotification object:nil];

 ...

- (void)didChangeOrientation:(NSNotification *)notification
{
    UIInterfaceOrientation orientation = [UIApplication sharedApplication].statusBarOrientation;

    if (UIInterfaceOrientationIsLandscape(orientation)) {
        NSLog(@"Landscape");
    }
    else {
        NSLog(@"Portrait");
    }
}

How do I round a double to two decimal places in Java?

double amount = 25.00;

NumberFormat formatter = new DecimalFormat("#0.00");

System.out.println(formatter.format(amount));

How do I set the value property in AngularJS' ng-options?

How the value attributes gets its value:

  • When using an array as datasource, it will be the index of the array element in each iteration;
  • When using an object as datasource, it will be the property name in each iteration.

So in your case it should be:

obj = { '1': '1st', '2': '2nd' };

<select ng-options="k as v for (k,v) in obj"></select>

.NET NewtonSoft JSON deserialize map to a different property name

Expanding Rentering.com's answer, in scenarios where a whole graph of many types is to be taken care of, and you're looking for a strongly typed solution, this class can help, see usage (fluent) below. It operates as either a black-list or white-list per type. A type cannot be both (Gist - also contains global ignore list).

public class PropertyFilterResolver : DefaultContractResolver
{
  const string _Err = "A type can be either in the include list or the ignore list.";
  Dictionary<Type, IEnumerable<string>> _IgnorePropertiesMap = new Dictionary<Type, IEnumerable<string>>();
  Dictionary<Type, IEnumerable<string>> _IncludePropertiesMap = new Dictionary<Type, IEnumerable<string>>();
  public PropertyFilterResolver SetIgnoredProperties<T>(params Expression<Func<T, object>>[] propertyAccessors)
  {
    if (propertyAccessors == null) return this;

    if (_IncludePropertiesMap.ContainsKey(typeof(T))) throw new ArgumentException(_Err);

    var properties = propertyAccessors.Select(GetPropertyName);
    _IgnorePropertiesMap[typeof(T)] = properties.ToArray();
    return this;
  }

  public PropertyFilterResolver SetIncludedProperties<T>(params Expression<Func<T, object>>[] propertyAccessors)
  {
    if (propertyAccessors == null)
      return this;

    if (_IgnorePropertiesMap.ContainsKey(typeof(T))) throw new ArgumentException(_Err);

    var properties = propertyAccessors.Select(GetPropertyName);
    _IncludePropertiesMap[typeof(T)] = properties.ToArray();
    return this;
  }

  protected override IList<JsonProperty> CreateProperties(Type type, MemberSerialization memberSerialization)
  {
    var properties = base.CreateProperties(type, memberSerialization);

    var isIgnoreList = _IgnorePropertiesMap.TryGetValue(type, out IEnumerable<string> map);
    if (!isIgnoreList && !_IncludePropertiesMap.TryGetValue(type, out map))
      return properties;

    Func<JsonProperty, bool> predicate = jp => map.Contains(jp.PropertyName) == !isIgnoreList;
    return properties.Where(predicate).ToArray();
  }

  string GetPropertyName<TSource, TProperty>(
  Expression<Func<TSource, TProperty>> propertyLambda)
  {
    if (!(propertyLambda.Body is MemberExpression member))
      throw new ArgumentException($"Expression '{propertyLambda}' refers to a method, not a property.");

    if (!(member.Member is PropertyInfo propInfo))
      throw new ArgumentException($"Expression '{propertyLambda}' refers to a field, not a property.");

    var type = typeof(TSource);
    if (!type.GetTypeInfo().IsAssignableFrom(propInfo.DeclaringType.GetTypeInfo()))
      throw new ArgumentException($"Expresion '{propertyLambda}' refers to a property that is not from type '{type}'.");

    return propInfo.Name;
  }
}

Usage:

var resolver = new PropertyFilterResolver()
  .SetIncludedProperties<User>(
    u => u.Id, 
    u => u.UnitId)
  .SetIgnoredProperties<Person>(
    r => r.Responders)
  .SetIncludedProperties<Blog>(
    b => b.Id)
  .Ignore(nameof(IChangeTracking.IsChanged)); //see gist

UUID max character length

Section 3 of RFC4122 provides the formal definition of UUID string representations. It's 36 characters (32 hex digits + 4 dashes).

Sounds like you need to figure out where the invalid 60-char IDs are coming from and decide 1) if you want to accept them, and 2) what the max length of those IDs might be based on whatever API is used to generate them.

How can I get the line number which threw exception?

Update to the answer

    // Get stack trace for the exception with source file information
    var st = new StackTrace(ex, true);
    // Get the top stack frame
    var frame = st.GetFrame(st.FrameCount-1);
    // Get the line number from the stack frame
    var line = frame.GetFileLineNumber();

Disable submit button on form submit

The following worked for me:

var form_enabled = true;
$().ready(function(){
       // allow the user to submit the form only once each time the page loads
       $('#form_id').on('submit', function(){
               if (form_enabled) {
                       form_enabled = false;
                       return true;
               }

               return false;
        });
});

This cancels the submit event if the user tries to submit the form multiple times (by clicking a submit button, pressing Enter, etc.)

How can I start PostgreSQL on Windows?

After a lot of search and tests i found the solution : if you are in windows :

1 - first you must found the PG databases directory execute the command as sql command in pgAdmin query tools

$ show data_directory;

result :

------------------------
- D:/PG_DATA/data      -
------------------------

2 - go to the bin directory of postgres in my case it's located "c:/programms/postgresSql/bin"

and open a command prompt (CMD) and execute this command :

pg_ctl -D "D:\PSG_SQL\data" restart

This should do it.

Getting attribute using XPath

If you are using PostgreSQL, this is the right way to get it. This is just an assumption where as you have a book table TITLE and PRICE column with populated data. Here's the query

SELECT xpath('/bookstore/book/title/@lang', xmlforest(book.title AS title, book.price AS price), ARRAY[ARRAY[]::TEXT[]]) FROM book LIMIT 1;

jQuery append and remove dynamic table row

There are multiple problems here

  1. Id should be unique in a page
  2. For dynamic elements, you need to use event delegation using .on()

Ex

$(document).ready(function(){
    $("#addCF").click(function(){
        $("#customFields").append('<tr valign="top"><th scope="row"><label for="customFieldName">Custom Field</label></th><td><input type="text" class="code" id="customFieldName" name="customFieldName[]" value="" placeholder="Input Name" /> &nbsp; <input type="text" class="code" id="customFieldValue" name="customFieldValue[]" value="" placeholder="Input Value" /> &nbsp; <a href="javascript:void(0);" id="remCF">Remove</a></td></tr>');
    });

    $("#customFields").on('click', '#remCF', function(){
        $(this).parent().parent().remove();
    });

});

Demo: Fiddle

See this demo where id properties are removed.

$(document).ready(function(){
    $("#addCF").click(function(){
        $("#customFields").append('<tr valign="top"><th scope="row"><label for="customFieldName">Custom Field</label></th><td><input type="text" class="code" name="customFieldName[]" value="" placeholder="Input Name" /> &nbsp; <input type="text" class="code" name="customFieldValue[]" value="" placeholder="Input Value" /> &nbsp; <a href="javascript:void(0);" class="remCF">Remove</a></td></tr>');
    });

    $("#customFields").on('click', '.remCF', function(){
        $(this).parent().parent().remove();
    });

});

dropping rows from dataframe based on a "not in" condition

You can use Series.isin:

df = df[~df.datecolumn.isin(a)]

While the error message suggests that all() or any() can be used, they are useful only when you want to reduce the result into a single Boolean value. That is however not what you are trying to do now, which is to test the membership of every values in the Series against the external list, and keep the results intact (i.e., a Boolean Series which will then be used to slice the original DataFrame).

You can read more about this in the Gotchas.

What does it mean with bug report captured in android tablet?

It's because you have turned on USB debugging in Developer Options. You can create a bug report by holding the power + both volume up and down.

Edit: This is what the forums say:

By pressing Volume up + Volume down + power button, you will feel a vibration after a second or so, that's when the bug reporting initiated.

To disable:

/system/bin/bugmailer.sh must be deleted/renamed.

There should be a folder on your SD card called "bug reports".

Have a look at this thread: http://forum.xda-developers.com/showthread.php?t=2252948

And this one: http://forum.xda-developers.com/showthread.php?t=1405639

How to make a drop down list in yii2?

There are some good solutions above, and mine is just a combination of two (I came here looking for a solution).

@Sarvar Nishonboyev's solution is good because it maintains the creation of the form input label and help-block for error messages.

I went with:

<?php
use yii\helpers\ArrayHelper;
use app\models\Product;
?>
<?=
$form->field($model, 'parent_id')
     ->dropDownList(
            ArrayHelper::map(Product::find()->asArray()->all(), 'parent_id', 'name')
            )
?>

Again, full credit to: @Sarvar Nishonboyev's and @ippi

How can I get the name of an object in Python?

Use a reverse dict.

fun_dict = {'fun1': fun1,
            'fun2': fun2,
            'fun3': fun3}

r_dict = dict(zip(fun_dict.values(), fun_dict.keys()))

The reverse dict will map each function reference to the exact name you gave it in fun_dict, which may or may not be the name you used when you defined the function. And, this technique generalizes to other objects, including integers.

For extra fun and insanity, you can store the forward and reverse values in the same dict. I wouldn't do that if you were mapping strings to strings, but if you are doing something like function references and strings, it's not too crazy.

How to execute a command prompt command from python

how about simply:

import os
os.system('dir c:\\')

Replace characters from a column of a data frame R

You can use the stringr library:

library('stringr')

a <- runif(10)
b <- letters[1:10]
c <- c(rep('A-B', 4), rep('A_B', 6))
data <- data.frame(a, b, c)

data

#             a b   c
# 1  0.19426707 a A-B
# 2  0.12902673 b A-B
# 3  0.78324955 c A-B
# 4  0.06469028 d A-B
# 5  0.34752264 e A_C
# 6  0.55313288 f A_C
# 7  0.31264280 g A_C
# 8  0.33759921 h A_C
# 9  0.72322599 i A_C
# 10 0.25223075 j A_C

data$c <- str_replace_all(data$c, '_', '-')

data

#             a b   c
# 1  0.19426707 a A-B
# 2  0.12902673 b A-B
# 3  0.78324955 c A-B
# 4  0.06469028 d A-B
# 5  0.34752264 e A-C
# 6  0.55313288 f A-C
# 7  0.31264280 g A-C
# 8  0.33759921 h A-C
# 9  0.72322599 i A-C
# 10 0.25223075 j A-C

Note that this does change factored variables into character.

How to find the foreach index?

Jonathan is correct. PHP arrays act as a map table mapping keys to values. in some cases you can get an index if your array is defined, such as

$var = array(2,5);

for ($i = 0; $i < count($var); $i++) {
    echo $var[$i]."\n";
}

your output will be

2
5

in which case each element in the array has a knowable index, but if you then do something like the following

$var = array_push($var,10);

for ($i = 0; $i < count($var); $i++) {
    echo $var[$i]."\n";
}

you get no output. This happens because arrays in PHP are not linear structures like they are in most languages. They are more like hash tables that may or may not have keys for all stored values. Hence foreach doesn't use indexes to crawl over them because they only have an index if the array is defined. If you need to have an index, make sure your arrays are fully defined before crawling over them, and use a for loop.

How to stop event bubbling on checkbox click

As others have mentioned, try stopPropagation().

And there is a second handler to try: event.cancelBubble = true; It's a IE specific handler, but it is supported in at least FF. Don't really know much about it, as I haven't used it myself, but it might be worth a shot, if all else fails.

How could I use requests in asyncio?

aiohttp can be used with HTTP proxy already:

import asyncio
import aiohttp


@asyncio.coroutine
def do_request():
    proxy_url = 'http://localhost:8118'  # your proxy address
    response = yield from aiohttp.request(
        'GET', 'http://google.com',
        proxy=proxy_url,
    )
    return response

loop = asyncio.get_event_loop()
loop.run_until_complete(do_request())

Remove all special characters, punctuation and spaces from string

import re
abc = "askhnl#$%askdjalsdk"
ddd = abc.replace("#$%","")
print (ddd)

and you shall see your result as

'askhnlaskdjalsdk

How to display a range input slider vertically

You can do this with css transforms, though be careful with container height/width. Also you may need to position it lower:

_x000D_
_x000D_
input[type="range"] {_x000D_
   position: absolute;_x000D_
   top: 40%;_x000D_
   transform: rotate(270deg);_x000D_
}
_x000D_
<input type="range"/>
_x000D_
_x000D_
_x000D_


or the 3d transform equivalent:

input[type="range"] {
   transform: rotateZ(270deg);
}

You can also use this to switch the direction of the slide by setting it to 180deg or 90deg for horizontal or vertical respectively.

How can I get href links from HTML using Python?

Simplest way for me:

from urlextract import URLExtract
from requests import get

url = "sample.com/samplepage/"
req = requests.get(url)
text = req.text
# or if you already have the html source:
# text = "This is html for ex <a href='http://google.com/'>Google</a> <a href='http://yahoo.com/'>Yahoo</a>"
text = text.replace(' ', '').replace('=','')
extractor = URLExtract()
print(extractor.find_urls(text))

output:

['http://google.com/', 'http://yahoo.com/']

How can I use grep to find a word inside a folder?

grep -nr search_string search_dir

will do a RECURSIVE (meaning the directory and all it's sub-directories) search for the search_string. (as correctly answered by usta).

The reason you were not getting any anwers with your friend's suggestion of:

grep -nr string

is because no directory was specified. If you are in the directory that you want to do the search in, you have to do the following:

grep -nr string .

It is important to include the '.' character, as this tells grep to search THIS directory.

jquery append div inside div with id and manipulate

It's just the wrong order

var e = $('<div style="display:block; float:left;width:'+width+'px; height:'+height+'px; margin-top:'+positionY+'px;margin-left:'+positionX+'px;border:1px dashed #CCCCCC;"></div>');
$('#box').append(e);    
e.attr('id', 'myid');

Append first and then access/set attr.

C++ Pass A String

Well, std::string is a class, const char * is a pointer. Those are two different things. It's easy to get from string to a pointer (since it typically contains one that it can just return), but for the other way, you need to create an object of type std::string.

My recommendation: Functions that take constant strings and don't modify them should always take const char * as an argument. That way, they will always work - with string literals as well as with std::string (via an implicit c_str()).

How to write to a JSON file in the correct format

To make this work on Ubuntu Linux:

  1. I installed the Ubuntu package ruby-json:

    apt-get install ruby-json
    
  2. I wrote the script in ${HOME}/rubybin/jsonDEMO

  3. $HOME/.bashrc included:

    ${HOME}/rubybin:${PATH}
    

(On this occasion I also typed the above on the bash command line.)

Then it worked when I entered on the command line:

jsonDemo

How to call multiple JavaScript functions in onclick event?

A link with 1 function defined

<a href="#" onclick="someFunc()">Click me To fire some functions</a>

Firing multiple functions from someFunc()

function someFunc() {
    showAlert();
    validate();
    anotherFunction();
    YetAnotherFunction();
}

How to view method information in Android Studio?

In addition to the answers here you might want to make sure that the documentation is downloaded. To do that go to SDK Manager

enter image description here

and then the SDK Tools tab and make sure Documentation for Android SDK is installed.

enter image description here

If it isn't check the box and click Apply.

selecting rows with id from another table

Try this (subquery):

SELECT * FROM terms WHERE id IN 
   (SELECT term_id FROM terms_relation WHERE taxonomy = "categ")

Or you can try this (JOIN):

SELECT t.* FROM terms AS t 
   INNER JOIN terms_relation AS tr 
   ON t.id = tr.term_id AND tr.taxonomy = "categ"

If you want to receive all fields from two tables:

SELECT t.id, t.name, t.slug, tr.description, tr.created_at, tr.updated_at 
  FROM terms AS t 
   INNER JOIN terms_relation AS tr 
   ON t.id = tr.term_id AND tr.taxonomy = "categ"

MAX() and MAX() OVER PARTITION BY produces error 3504 in Teradata Query

Logically OLAP functions are calculated after GROUP BY/HAVING, so you can only access columns in GROUP BY or columns with an aggregate function. Following looks strange, but is Standard SQL:

SELECT employee_number,
       MAX(MAX(course_completion_date)) 
           OVER (PARTITION BY course_code) AS max_course_date,
       MAX(course_completion_date) AS max_date
FROM employee_course_completion
WHERE course_code IN ('M910303', 'M91301R', 'M91301P')
GROUP BY employee_number, course_code

And as Teradata allows re-using an alias this also works:

SELECT employee_number,
       MAX(max_date) 
           OVER (PARTITION BY course_code) AS max_course_date,
       MAX(course_completion_date) AS max_date
FROM employee_course_completion
WHERE course_code IN ('M910303', 'M91301R', 'M91301P')
GROUP BY employee_number, course_code

How we can bold only the name in table td tag not the value

Surround what you want to be bold with:

<span style="font-weight:bold">Your bold text</span>

This would go inside your <td> tag.

How to set the value of a hidden field from a controller in mvc

Please try using following way.

@Html.Hidden("hdnFlag",(object) Convert.ToInt32(ViewBag.page_Count))

How to use protractor to check if an element is visible?

I had a similar issue, in that I only wanted return elements that were visible in a page object. I found that I'm able to use the css :not. In the case of this issue, this should do you...

expect($('i.icon-spinner:not(.ng-hide)').isDisplayed()).toBeTruthy();

In the context of a page object, you can get ONLY those elements that are visible in this way as well. Eg. given a page with multiple items, where only some are visible, you can use:

this.visibileIcons = $$('i.icon:not(.ng-hide)'); 

This will return you all visible i.icons

How to check the maximum number of allowed connections to an Oracle database?

The sessions parameter is derived from the processes parameter and changes accordingly when you change the number of max processes. See the Oracle docs for further info.

To get only the info about the sessions:

    select current_utilization, limit_value 
    from v$resource_limit 
    where resource_name='sessions';
CURRENT_UTILIZATION LIMIT_VALUE
------------------- -----------
                110         792

Try this to show info about both:

    select resource_name, current_utilization, max_utilization, limit_value 
    from v$resource_limit 
    where resource_name in ('sessions', 'processes');
RESOURCE_NAME CURRENT_UTILIZATION MAX_UTILIZATION LIMIT_VALUE
------------- ------------------- --------------- -----------
processes                      96             309         500
sessions                      104             323         792

How to do logging in React Native?

Chrome Devtool is the best and easiest way for logging and debugging.

PHP Error: Function name must be a string

A useful explanation to how braces are used (in addition to Filip Ekberg's useful answer, above) can be found in the short paper Parenthesis in Programming Languages.

Is ConfigurationManager.AppSettings available in .NET Core 2.0?

You can use Configuration to resolve this.

Ex (Startup.cs):

You can pass by DI to the controllers after this implementation.

public class Startup
{
    public Startup(IHostingEnvironment env)
    {
        var builder = new ConfigurationBuilder()
        .SetBasePath(env.ContentRootPath)
        .AddJsonFile("appsettings.json", optional: true, reloadOnChange: true);

        Configuration = builder.Build();

    }

    public IConfiguration Configuration { get; }

    // This method gets called by the runtime. Use this method to add services to the container.
    public void ConfigureServices(IServiceCollection services)
    {

        var microserviceName = Configuration["microserviceName"];

       services.AddSingleton(Configuration);

       ...
    }

groovy.lang.MissingPropertyException: No such property: jenkins for class: groovy.lang.Binding

For me this problem occurred because I had a some invalid character in my Groovy script. In our case this was an extra blank line after the closing bracket of the script.

Android TabLayout Android Design

I've just managed to setup new TabLayout, so here are the quick steps to do this (????)?*:???

  1. Add dependencies inside your build.gradle file:

    dependencies {
        compile 'com.android.support:design:23.1.1'
    }
    
  2. Add TabLayout inside your layout

    <?xml version="1.0" encoding="utf-8"?>
    <LinearLayout xmlns:android="http://schemas.android.com/apk/res/android"
          android:layout_width="match_parent"
          android:layout_height="match_parent"
          android:orientation="vertical">
    
        <android.support.v7.widget.Toolbar
            android:id="@+id/toolbar"
            android:layout_width="match_parent"
            android:layout_height="wrap_content"
            android:background="?attr/colorPrimary"/>
    
        <android.support.design.widget.TabLayout
            android:id="@+id/tab_layout"
            android:layout_width="match_parent"
            android:layout_height="wrap_content"/>
    
        <android.support.v4.view.ViewPager
            android:id="@+id/pager"
            android:layout_width="match_parent"
            android:layout_height="match_parent"/>
    
    </LinearLayout>
    
  3. Setup your Activity like this:

    import android.os.Bundle;
    import android.support.design.widget.TabLayout;
    import android.support.v4.app.Fragment;
    import android.support.v4.app.FragmentManager;
    import android.support.v4.app.FragmentPagerAdapter;
    import android.support.v4.view.ViewPager;
    import android.support.v7.app.AppCompatActivity;
    import android.support.v7.widget.Toolbar;
    
    public class TabLayoutActivity extends AppCompatActivity {
    
        @Override
        protected void onCreate(Bundle savedInstanceState) {
            super.onCreate(savedInstanceState);
            setContentView(R.layout.activity_pull_to_refresh);
    
            Toolbar toolbar = (Toolbar) findViewById(R.id.toolbar);
            TabLayout tabLayout = (TabLayout) findViewById(R.id.tab_layout);
            ViewPager viewPager = (ViewPager) findViewById(R.id.pager);
    
            if (toolbar != null) {
                setSupportActionBar(toolbar);
            }
    
            viewPager.setAdapter(new SectionPagerAdapter(getSupportFragmentManager()));
            tabLayout.setupWithViewPager(viewPager);
        }
    
        public class SectionPagerAdapter extends FragmentPagerAdapter {
    
            public SectionPagerAdapter(FragmentManager fm) {
                super(fm);
            }
    
            @Override
            public Fragment getItem(int position) {
                switch (position) {
                    case 0:
                        return new FirstTabFragment();
                    case 1:
                    default:
                        return new SecondTabFragment();
                }
            }
    
            @Override
            public int getCount() {
                return 2;
            }
    
            @Override
            public CharSequence getPageTitle(int position) {
                switch (position) {
                    case 0:
                        return "First Tab";
                    case 1:
                    default:
                        return "Second Tab";
                }
            }
        }
    
    }
    

Which loop is faster, while or for?

As others have said, any compiler worth its salt will generate practically identical code. Any difference in performance is negligible - you are micro-optimizing.

The real question is, what is more readable? And that's the for loop (at least IMHO).

Oracle copy data to another table

insert into EXCEPTION_CODES (CODE, MESSAGE)
select CODE, MESSAGE from Exception_code_tmp

Import multiple csv files into pandas and concatenate into one DataFrame

This is how you can do using Colab on Google Drive

import pandas as pd
import glob

path = r'/content/drive/My Drive/data/actual/comments_only' # use your path
all_files = glob.glob(path + "/*.csv")

li = []

for filename in all_files:
    df = pd.read_csv(filename, index_col=None, header=0)
    li.append(df)

frame = pd.concat(li, axis=0, ignore_index=True,sort=True)
frame.to_csv('/content/drive/onefile.csv')

Base64 encoding and decoding in client-side Javascript

Here is a tightened up version of Sniper's post. It presumes well formed base64 string with no carriage returns. This version eliminates a couple of loops, adds the &0xff fix from Yaroslav, eliminates trailing nulls, plus a bit of code golf.

decodeBase64 = function(s) {
    var e={},i,b=0,c,x,l=0,a,r='',w=String.fromCharCode,L=s.length;
    var A="ABCDEFGHIJKLMNOPQRSTUVWXYZabcdefghijklmnopqrstuvwxyz0123456789+/";
    for(i=0;i<64;i++){e[A.charAt(i)]=i;}
    for(x=0;x<L;x++){
        c=e[s.charAt(x)];b=(b<<6)+c;l+=6;
        while(l>=8){((a=(b>>>(l-=8))&0xff)||(x<(L-2)))&&(r+=w(a));}
    }
    return r;
};

send mail to multiple receiver with HTML mailto

"There are no safe means of assigning multiple recipients to a single mailto: link via HTML. There are safe, non-HTML, ways of assigning multiple recipients from a mailto: link."

http://www.sightspecific.com/~mosh/www_faq/multrec.html

For a quick fix to your problem, change your ; to a comma , and eliminate the spaces between email addresses

<a href='mailto:[email protected],[email protected]'>Email Us</a>

How to get JSON Key and Value?

First, I see you're using an explicit $.parseJSON(). If that's because you're manually serializing JSON on the server-side, don't. ASP.NET will automatically JSON-serialize your method's return value and jQuery will automatically deserialize it for you too.

To iterate through the first item in the array you've got there, use code like this:

var firstItem = response.d[0];

for(key in firstItem) {
  console.log(key + ':' + firstItem[key]);
}

If there's more than one item (it's hard to tell from that screenshot), then you can loop over response.d and then use this code inside that outer loop.

Where is android studio building my .apk file?

Mine application's apk was at this location

C:\Users\haseeb_mir\AndroidStudioProjects\MyTestApp\app\build\outputs\apk\debug

How to set a maximum execution time for a mysql query?

pt_kill has an option for such. But it is on-demand, not continually monitoring. It does what @Rafa suggested. However see --sentinel for a hint of how to come close with cron.